85
Министерство образования и науки Российской Федерации Федеральное государственное бюджетное образовательное учреждение высшего профессионального образования «Амурский государственный университет» Кафедра математического анализа и моделирования УЧЕБНО-МЕТОДИЧЕСКИЙ КОМПЛЕКС ДИСЦИПЛИНЫ ГЕОМЕТРИЯ И АЛГЕБРА Основной образовательной программы по специальности подготовки 010500.62 «Прикладная математика и информатика» Благовещенск 2012

irbis.amursu.ru · 4 1. J : ; H Q :Я I J H = J : F F : ГЕОМЕТРИЯ И АЛГЕБРА Направление подготовки 010500.62 «Прикладная математика

  • Upload
    others

  • View
    25

  • Download
    0

Embed Size (px)

Citation preview

Page 1: irbis.amursu.ru · 4 1. J : ; H Q :Я I J H = J : F F : ГЕОМЕТРИЯ И АЛГЕБРА Направление подготовки 010500.62 «Прикладная математика

Министерство образования и науки Российской Федерации

Федеральное государственное бюджетное образовательное учреждение

высшего профессионального образования

«Амурский государственный университет»

Кафедра математического анализа и моделирования

УЧЕБНО-МЕТОДИЧЕСКИЙ КОМПЛЕКС ДИСЦИПЛИНЫ

ГЕОМЕТРИЯ И АЛГЕБРА

Основной образовательной программы по специальности подготовки

010500.62 «Прикладная математика и информатика»

Благовещенск 2012

Page 2: irbis.amursu.ru · 4 1. J : ; H Q :Я I J H = J : F F : ГЕОМЕТРИЯ И АЛГЕБРА Направление подготовки 010500.62 «Прикладная математика

2

УМКД разработан канд. техн. наук, доцентом

Кван Натальей Владимировной,

Рассмотрен и рекомендован на заседании кафедры МАиМ

Протокол заседания кафедры от «___» ___________ 2012г. №__

И.о. зав. кафедрой ______________________ /Сельвинский В.В./

УТВЕРЖДЕН

Протокол заседания УМСС направления подготовки 010500.62 «прикладная

математика и информатика»

от «___» __________ 2012г. №___

Председатель УМСС _________________ / Сельвинский В.В./

Page 3: irbis.amursu.ru · 4 1. J : ; H Q :Я I J H = J : F F : ГЕОМЕТРИЯ И АЛГЕБРА Направление подготовки 010500.62 «Прикладная математика

3

СОДЕРЖАНИЕ

1. РАБОЧАЯ ПРОГРАММА 4 стр.

2. КРАТКОЕ ИЗЛОЖЕНИЕ ПРОГРАММНОГО МАТЕРИАЛА 20 стр.

3. МЕТОДИЧЕСКИЕ УКАЗАНИЯ ДЛЯ ПРОВЕДЕНИЯ ПРАКТИЧЕСКИХ ЗАНЯТИЙ 49 стр.

4. МЕТОДИЧЕСКИЕ УКАЗАНИЯ ПО САМОСТОЯТЕЛЬНОЙ РАБОТЕ СТУДЕНТОВ 72 стр.

5. КОНТРОЛЬ ЗНАНИЙ 78 стр.

6. ИТОГОВЫЙ КОНТРОЛЬ (экзамен) 82 стр.

Page 4: irbis.amursu.ru · 4 1. J : ; H Q :Я I J H = J : F F : ГЕОМЕТРИЯ И АЛГЕБРА Направление подготовки 010500.62 «Прикладная математика

4

1. РАБОЧАЯ ПРОГРАММА

ГЕОМЕТРИЯ И АЛГЕБРА

Направление подготовки 010500.62 «Прикладная математика и информатика»

Степень выпускника бакалавр прикладной математики и информатики

Курс 1 Семестр 1, 2

Лекции 144 часа (72+72)

Экзамен 1, 2 семестры

Практические (семинарские) занятия 108 часов (54+54)

Самостоятельная работа 105 часов, в том числе 54 часа на экзамен

Общая трудоемкость дисциплины 357 часов, 11 з.е.

Составитель Кван Наталья Владимировна, доцент каф. МАиМ, к. техн.н.

Факультет математики и информатики

Кафедра математического анализа и моделирования

1.1.ЦЕЛИ И ЗАДАЧИ ОСВОЕНИЯ ДИСЦИПЛИНЫ

Целями освоения дисциплины являются:

получение базовых знаний, умений и навыков по алгебре и геометрии, а также

формирование общекультурных и профессиональных компетенций, необходимых для

успешного профессиональной деятельности будущих специалистов; Достижение этих

целей требует решения следующих задач:

изучение базовых понятий аналитической геометрии и линейной алгебры;

освоение основных приемов решения практических задач по темам дисциплины;

приобретение опыта построения математических моделей различных явлений и

проведения необходимых расчѐтов в рамках построенных моделей; употребления

математической символики для выражения количественных и качественных

отношений объектов;

подготовка к поиску и анализу профильной научно-технической информации,

необходимой для решения конкретных научно-исследовательских и прикладных

задач, в том числе при выполнении междисциплинарных проектов;

привитие общематематической культуры: умение логически мыслить, проводить

доказательства основных утверждений, устанавливать логические связи между

понятиями;

формирование социально-личностных качеств студентов: целеустремленности,

организованности, трудолюбия, коммуникативности, готовности к деятельности в

профессиональной среде, ответственности за принятие профессиональных

решений.

1.2.МЕСТО ДИСЦИПЛИНЫ В СТРУКТУРЕ ООП ВПО

Дисциплина «Геометрия и алгебра» входит в базовую часть математического и

естественно-научного цикла ЕН.Ф.1.2 ООП ВПО.

Понятия аналитической геометрии и линейной алгебры, алгебраические и

аналитические методы исследования непосредственно и опосредованно проникли во

многие разделы естествознания, пронизывают все фундаментальные общематематические

курсы. Методы аналитической геометрии и линейной алгебры имеют универсальное

значение.

Page 5: irbis.amursu.ru · 4 1. J : ; H Q :Я I J H = J : F F : ГЕОМЕТРИЯ И АЛГЕБРА Направление подготовки 010500.62 «Прикладная математика

5

1.3. ТРЕБОВАНИЯ К УРОВНЮ ОСВОЕНИЯ СОДЕРЖАНИЯ ДИСЦИПЛИНЫ

В результате освоения дисциплины обучающийся должен демонстрировать следующие

результаты образования:

1) Знать:

– основные понятия и результаты по алгебре (теория матриц, системы линейных

уравнений, теория многочленов, линейные пространства и линейная зависимость,

собственные векторы и собственные значения, канонический вид матриц линейных

операторов, геометрия метрических линейных пространств, свойства билинейных

функций, классификацию квадрик, основы теории групп и колец). Студенты должны

знать логические связи между ними;

– основные понятия геометрии, определения и свойства математических объектов в этой

области, формулировки утверждений, методы их доказательства, возможные сферы их

приложений, в том числе в компьютерном моделировании геометрических объектов и

явлений.

2) Уметь:

– решать системы линейных уравнений, вычислять определители, исследовать свойства

многочленов, находить собственные векторы и собственные значения, канонический вид

матриц линейных операторов, классифицировать квадрики, основные свойства групп,

колец;

– решать задачи вычислительного и теоретического характера в области геометрии

трехмерного евклидова (аффинного) пространства, доказывать утверждения.

3) Владеть:

– математическим аппаратом алгебры и геометрии, аналитическими методами

исследования алгебраических и геометрических объектов.

1.4.СТРУКТУРА И СОДЕРЖАНИЕ ДИСЦИПЛИНЫ «ГЕОМЕТРИЯ И АЛГЕБРА»

Общая трудоемкость дисциплины составляет 11 зачетных единиц, 411 часов.

п/

п Раздел

дисциплины

Сем

естр

Нед

еля с

емес

тра

Виды учебной

работы, включая

самостоятельную

работу студентов

и трудоемкость

(в часах)

Формы

текущего

контроля

успеваемости

(по неделям

семестра)

Форма

промежуточно

й аттестации

(по семестрам)

Лек

-

ции

Пра

к.

Заня

-тия

Сам.

рабо

та

1 Алгебраические структуры 1 1, 2,

3

6 12 4 УО, ПДЗ, МД,

СР, ТС, РГР

№1

2 Теория определителей 1 4, 5 4 4 4 УО, ПДЗ, МД,

СР

3 Матрицы 1 6, 7 4 6 4 УО, ПДЗ, МД,

СР, КР№1, ТС

Page 6: irbis.amursu.ru · 4 1. J : ; H Q :Я I J H = J : F F : ГЕОМЕТРИЯ И АЛГЕБРА Направление подготовки 010500.62 «Прикладная математика

6

4 Арифметическое n-мерное

векторное пространство.

Системы линейных уравнений.

1 8, 9 4 8 6 УО, ПДЗ, МД,

СР, ТС

5 Векторные пространства. 1 10,

11

4 4 4 УО, ПДЗ, МД,

СР, РГР №2,

КЛ

6 Векторы на плоскости и в

пространстве

1 12,

13

4 8 4 УО, ПДЗ, МД,

СР, ТС

7 Прямая линия на плоскости 1 14,

15

4 6 4 УО, ПДЗ, МД,

СР, ТС, КР №2

8 Линии второго порядка 1 16,

17

4 6 6 УО, ПДЗ, МД,

СР, РГР №3,

ТС

9 Итоговое повторение 1 18 2 6 Экзамен

10 Плоскость 2 1, 2 4 6 4 УО, ПДЗ, МД,

СР, ТС

11 Прямая в пространстве 2 3, 4 4 6 4 УО, ПДЗ, МД,

СР, ТС, КР№3

12 Поверхности второго порядка 2 5, 6,7 6 9 6 УО, ПДЗ, МД,

СР РГР №4,

13 Теория многочленов 8, 9 4 6 4 УО, ПДЗ, СР,

КР

14 Линейные операторы 2 10,

11,

12

6 9 6 УО, ПДЗ, МД,

СР,

15 Евклидовы пространства 2 13,

14,

15

6 8 6 УО, ПДЗ, МД,

СР, КЛ, РГР

№5

16 Квадратичные формы и квадрики 2 16,

17

4 8 4 УО, ПДЗ, МД,

СР, ТС, КР№4

17 Итоговое повторение 2 18 2 2 3 Экзамен

УО – устный опрос, ПДЗ – проверка домашнего задания, МД – математический диктант, СР

– самостоятельная работа, ТС – тестирование, КР – контрольная работа, КЛ – коллоквиум,

ДЗ – домашнее задание, КП – конспект, РГР – расчетно-графическая работа.

КР№1 – «Матрицы, определители, систмемы линейных урвнений»,

КР№2 – «Векторы. Прямая линия на плоскости»,

КР№3 – «Плоскость и прямая в пространстве»,

КР№4 – «Квадратичные формы и квадрики».

РГР№1 – «Алгебраические структуры. Комплексные числа»,

Page 7: irbis.amursu.ru · 4 1. J : ; H Q :Я I J H = J : F F : ГЕОМЕТРИЯ И АЛГЕБРА Направление подготовки 010500.62 «Прикладная математика

7

РГР№2 – «Матрицы и определители»,

РГР№3 – «Аналитическая геометрия на плоскости»,

РГР№4 – «Аналитическая геометрия в пространстве»

1.5. СОДЕРЖАНИЕ РАЗДЕЛОВ И ТЕМ ДИСЦИПЛИНЫ

5.1. Лекции.

1 семестр

Лекция 1. Числовые множества. Бинарная алгебраическая операция. Примеры.

Свойства.

Лекция 2. Группа. Кольцо. Поле. Примеры.

Лекция 3. Построение поля комплексных чисел.

Лекция 4.Действия над комплексными числами в алгебраической форме.

Лекция 5. Тригонометрическая форма комплексного числа. Действия в

тригонометрической форме. Геометрическая интерпретация действий. Показательная

форма.

Лекция 6. Перестановки. Инверсия. Группа перестановок. Четность подстановок.

Знакопеременная группа.

Лекция 7. Определитель n-го порядка. Определитель 2 и 3 порядков. Свойства

определителя.

Лекция 8. Теорема Безу. Теорема Вандермонда. Теорема Лапласа.

Лекция 9. Виды матриц. Действия над матрицами. Свойства действий. Обратная

матрица.

Лекция 10. Формула для вычисления обратной матрицы. Вывод формул Крамера.

Лекция 111 - 12. Арифметическое векторное пространство. Линейная зависимость и

независимость векторов. Свойства линейной зависимости.

Лекция 13. Теоремы о линейной зависимости векторов.

Лекция 14. Базис и ранг системы векторов. Ранг матрицы.

Лекция 15. Системы линейных уравнений. Способы записи. Критерий Кронеккера –

Капелли.

Лекция 16. Векторные пространства. Примеры. Свойства.

Лекция 17. Линейные подпространства. Критерий подпространства. Линейная

оболочка.

Лекция 18 - 19. Сумма и пересечение подпространств. Пространство решений

системы линейных однородных уравнений. Фундаментальная система решений. Линейное

многообразие.

Лекция 20. Векторы на плоскости и в пространcтве. Сложение и умножение вектора

на число. Коллинеарные и компланарные векторы.

Лекция 21. Декартова и полярная системы координат. Координаты векторов.

Действия над векторами в координата.

Лекция 22. Скалярное произведение векторов.

Лекция 23 - 24. Векторное и смешанное произведения векторов.

Лекция 25. Простейшие задачи аналитической геометрии.

Лекция 26 - 27. Прямая линия на плоскости. Общее уравнение прямой. Уравнение

прямой с угловым коэффициентом. Уравнение «в отрезках». Нормальное уравнение.

Лекция 28. Взаимное расположение прямых на плоскости. Угол между прямыми.

Расстояние от точки до прямой.

Лекция 29. Эллипс. Геометрические свойства эллипса.

Лекция 30. Гипербола. Геометрические свойства гиперболы.

Лекция 31. Парабола и ее геометрические свойства.

Лекция 32. Полярные уравнения линий 2 порядка.

Page 8: irbis.amursu.ru · 4 1. J : ; H Q :Я I J H = J : F F : ГЕОМЕТРИЯ И АЛГЕБРА Направление подготовки 010500.62 «Прикладная математика

8

Лекция 33. Группа. Примеры. Подгруппа. Примеры. Смежные классы. Разложение

группы по подгруппе.

Лекция 34. Нормальные делители группы. Фактор – группа.

Лекция 35. Изоморфизм и гомоморфизм групп. Теорема о гомомоморфизме.

Лекция 36. Обобщение пройденного материала.

2 семестр.

Лекция 1. Плоскость. Общее уравнение. Взаимное расположение плоскостей.

Лекция 2. Угол между плоскостями. Расстояние от точки до плоскости.

Лекция 3. Прямая линия в пространстве. Виды уравнений прямой.

Лекция 4. Взаимное расположение прямых. Взаимное расположение прямых в

пространстве.

Лекция 5 - 6. Поверхности 2-го порядка. Эллипсоиды и гиперболоиды.

Лекция 7 - 9. Параболоиды. Цилиндр. Конус.

Лекция 10. Пересечение поверхностей. Классификация поверхностей 2-го порядков.

Лекция 11. Многочлены над областью целостности. Теорема Безу. Применение

семы Горнера.

Лекция 12. Многочлены над полем. НОД и НОК многочленов. Алгоритм Евклида.

Лекция 13. Неприводимые и приводимые многочлены над данным полем.

Выделение кратных множителей.

Лекция 14 - 16. Многочлены над полем C, R, Q.

Лекция 17. Уравнения 3 и 4 степени.

Лекция 18 - 19. Линейные операторы. Примеры. Свойства. Матрица линейного

оператора. Формула матрицы линейного оператора при изменении базиса.

Лекция 20. Обратное преобразование. Вырожденные и невырожденные

преобразования.

Лекция 21. Образ, ранг, ядро и дефект линейного оператора.

Лекция 22 - 23. Собственные векторы и собственные значения линейного оператора.

Собственные значения матрицы линейного оператора с симметрической матрицы.

Диагональная форма матрицы.

Лекция 24. Евклидовы пространства. Примеры. Свойства. Длина вектора. Угол

между векторами. Неравенства Коши – Буняковского.

Лекция 25 - 26. Ортогональность векторов. Ортонормированный базис.

Ортогональное дополнение. Процесс ортогонализации. Ортогональные матрицы.

Лекция 27 - 28. Ортогональная проекция и ортогональная составляющая вектора.

Определитель Грамма. Расстояние от вектора до многообразия. Угол между вектором и

подпространством.

Лекция 29 - 30. Квадратичные формы. Матрица квадратичной формы. Канонический

вид квадратичной формы. Метод Лагранжа приведения к каноническому виду.

Лекция 31. Метод ортогонального преобразования квадратичной формы к

каноническому виду. Закон инерции.

Лекция 32 – 35. Квадрики. Классификация квадрик в аффинном и евклидовом

пространстве. Приведение квадрик к каноническому виду.

Лекция 36. Итоговое повторение.

5.2. Практические (лабораторные/семинарские) занятия.

1 семестр

Практическое занятие 1. Определители 2 и 3 порядков. Линейные уравнения с 2 и 3

неизвестными. Самостоятельная работа «Решение линейных уравнений».

Практическое занятие 2. Математический диктант «Числовые множества». Решение

задач на определение свойств БАО.

Практическое занятие 3. Самостоятельная работа «Свойства БАО». Решение задач

на определение группы, кольца и поля.

Page 9: irbis.amursu.ru · 4 1. J : ; H Q :Я I J H = J : F F : ГЕОМЕТРИЯ И АЛГЕБРА Направление подготовки 010500.62 «Прикладная математика

9

Практическое занятие 4. Решение задач по теме «Алгебраическая форма

комплексного числа». Самостоятельная работа «Действия над комплексными числами в

алгебраической форме».

Практическое занятие 5. Решение задач по теме «Тригонометрическая форма

комплексного числа». Самостоятельная работа «Действия над комплексными числами в

тригонометрической форме».

Практическое занятие 6. Изображение комплексных чисел и действий над ними на

плоскости. Контрольная работа № 1.

Практическое занятие 7. Математический диктант «Подстановки, перестановки,

свойства определителей». Действия с подстановками. Вычисление определителей

треугольным методом и с помощью разложения по строке или столбцу.

Практическое занятие 8. Самостоятельная работа «Вычисление определителей».

Решение задач по темам «Применение теоремы Лапласа» и «Вычисление определителей n

–го порядка».

Практическое занятие 9. Решение уравнений по формулам Крамера.

Практическое занятие 10. Действия над матрицами. Вычисление обратной матрицы

методом приписывания единичной.

Практическое занятие 11 Вычисление обратной матрицы по формуле. Решение

матричных уравнений. Самостоятельная работа «Решение уравнений по формулам

Крамера и матричным методом».

Практическое занятие 12. Решение систем линейных уравнений методом Гаусса.

Самостоятельная работа «Решение уравнений по формулам Крамера, методом Гаусса и

матричным методом».

Практическое занятие 13. Математический диктант «Свойства линейной

зависимости и независимости векторов». Линейная зависимость и независимость

векторов. Базис и ранг системы векторов. Нахождение ранга матрицы.

Практическое занятие 14.Исследование систем линейных уравнений.

Самостоятельная работа «Системы линейных уравнений»

Практическое занятие 15. Контрольная работа № 2.

Практическое занятие 16. Математический диктант «Векторные пространства».

Решение задач по теме «Векторные пространства».

Практическое занятие 17. Нахождение фундаментального решения системы

линейных однородных уравнений и линейного многообразия системы линейных

неоднородных уравнений.

Практическое занятие 18. Действия над векторами. Решение задач геометрии

векторным методом. Тестирование по теме «Векторы»

Практическое занятие 19. Решение задач по теме «Скалярное произведение

векторов». Самостоятельная работа.

Практическое занятие 20. Математический диктант «Скалярное, векторное и

смешанное произведение векторов». Решение задач по теме «Векторное и смешанное

произведение векторов».

Практическое занятие 21. Контрольная работа № 3 «Векторы».

Практическое занятие 22. Решение задач по теме «Прямая линия на плоскости».

Практическое занятие 23. Математический диктант «Виды уравнений прямой.

Взаимное расположение прямых». Решение задач по теме «Прямая линия на плоскости».

Практическое занятие 24. Решение задач по теме «Прямая линия на плоскости».

Самостоятельная работа.

Практическое занятие 25. Математический диктант «Линии второго порядка».

Решение задач на геометрические свойства эллипса. Построение эллипса.

Практическое занятие 26. Решение задач на геометрические свойства гиперболы и

параболы». Построение гиперболы и параболы. Тестирование «Линии второго порядка».

Page 10: irbis.amursu.ru · 4 1. J : ; H Q :Я I J H = J : F F : ГЕОМЕТРИЯ И АЛГЕБРА Направление подготовки 010500.62 «Прикладная математика

10

Практическое занятие 27. Контрольная работа № 4 «Прямая линия. Линии второго

порядка».

2 семестр

Практическое занятие 1. Математический диктант. Решение задач по теме

«Плоскость».

Практическое занятие 2. Решение задач по теме «Плоскость».

Практическое занятие 3. Решение задач по теме «Плоскость». Самостоятельная

работа.

Практическое занятие 4. Математический диктант. Решение задач по теме «Прямая

линия».

Практическое занятие 5. Решение задач по теме «Взаимное расположение прямых в

пространстве».

Практическое занятие 6. Математический диктант. Решение задач по теме

«Взаимное расположение прямых и плоскостей в пространстве». Самостоятельная работа.

Практическое занятие 7. Построение эллипсоида, гиперболоидов.

Практическое занятие 8. Построение параболоидов.

Практическое занятие 9. Математический диктант. Построение конусов и

цилиндров.

Практическое занятие 10. Тестирование «Поверхности 2 порядка». Построение

пересечения поверхностей.

Практическое занятие 11. Контрольная работа по теме «Аналитическая геометрия в

пространстве».

Практическое занятие 12. Решение задач по теме «Многочлены над областью

целостности и полем».

Практическое занятие 13. Решение уравнений 3 и 4 степени.

Практическое занятие 14. Решение задач по теме «Многочлены над полями С и Q».

Самостоятельная работа.

Практическое занятие 14. Решение задач по теме «Линейные операторы».

Практическое занятие 15. Решение задач по теме «Линейные операторы».

Практическое занятие 16. Решение задач по теме «Линейные операторы».

Практическое занятие 17. Решение задач по теме «Линейные операторы».

Практическое занятие 18. Решение задач по теме «Линейные операторы».

Тестирование.

Практическое занятие 19. Решение задач по теме «Евклидовы пространства».

Практическое занятие 20. Контрольная работа по теме «Линейные операторы».

Практическое занятие 21. Решение задач по теме «Евклидовы пространства».

Практическое занятие 22. Решение задач по теме «Евклидовы пространства».

Практическое занятие 23. Решение задач по теме «Евклидовы пространства».

Практическое занятие 24. Решение задач по теме «Приведение квадратичной формы

к каноническому виду методом Лагранжа и методом Якоби».

Практическое занятие 25. Решение задач по теме «Приведение квадратичной формы

к каноническому виду методом ортогонального преобразования».

Практическое занятие 26. Решение задач по теме «Приведение квадрики к

каноническому виду в евклидовом пространстве».

Практическое занятие 27. Контрольная работа «Евклидовы пространства.

Квадратичные формы».

1.6. САМОСТОЯТЕЛЬНАЯ РАБОТА

п/п

№ раздела

(темы)

дисциплины

Форма (вид)

самостоятельной работы

Трудоѐмко

сть в часах

Page 11: irbis.amursu.ru · 4 1. J : ; H Q :Я I J H = J : F F : ГЕОМЕТРИЯ И АЛГЕБРА Направление подготовки 010500.62 «Прикладная математика

11

1 семестр

1 1 ИЗ «БАО. Группа» 3

2 1 КП «Кольцо. Поле» 2

3 1 КП «Геометрич. интерпретация действий над

компл. числами»

2

4 1 ИЗ «Комплексные числа» 3

5 2 КП «Свойства определителя» 2

6 2 КП «Теорема Лапласа» 2

7 2 ИЗ «Вычисление определителей» 3

8 3 КП «Матрицы» 2

9 3 ИЗ «Матрицы» 4

10 4 КП «Свойства линейной зависимости» 2

11 4 ИЗ «Исследование систем уравнений» 3

12 4 ИЗ «Системы уравнений» 4

13 5 ИЗ «Подпространства векторных пространств» 3

14 5 КП «Подпространства» 2

15 5 ИЗ «Векторные пространства» 4

16 6 ИЗ «Векторы на плоскости» 4

17 6 ИЗ «Скалярное произведение векторов» 3

18 7 КП « Уравнения прямой» 2

19 7 КП «Полярное уравнение прямой» 2

20 7 ИЗ «Прямая на плоскости» 3

21 8 КП «Свойства эллипса и гиперболы» 2

22 8 КП «Свойства параболы» 2

23 8 ИЗ «Линии второго порядка» 4

Итого за 1 семестр 63

Экзамен 45

2 семестр

1 10 КП «Нормальное уравнение плоскости.

Расстояние от точки до плоскости»

1

2 11 ИЗ «Прямая и плоскость в пространстве» 3

3 12 ИЗ «Поверхности второго порядка» 3

4 13 ИЗ «Действия с многочленами» 2

5 13 КП «Решение уравнений 4 степени» 2

6 13 ИЗ «Многочлены» 3

7 14 КП « Действия над линейными операторами» 2

8 14 КП «Инвариантные подпространства и

индуцированные преобразования»

2

9 14 КП «Приведение матрицы к диагональной

форме»

1

10 14 ИЗ «Линейные операторы» 3

11 15 КП «Неравенство треугольника. Метрические

пространства»

2

12 15 КП «Ортогональное дополнение» 2

13 15 ИЗ «Евклидовы пространства» 3

14 16 КП «Метод Якоби. Критерий Сельверстра» 1

15 16 ИЗ «Приведение квадратичных форм к

каноническому виду»

3

16 16 КП «Классификация квадрик в евклидовом

пространстве»

2

17 17 КП «Векторные пространства над полем С» 2

Page 12: irbis.amursu.ru · 4 1. J : ; H Q :Я I J H = J : F F : ГЕОМЕТРИЯ И АЛГЕБРА Направление подготовки 010500.62 «Прикладная математика

12

Итого в семестре 36

Экзамен 36

ВСЕГО 180

1.7.ОБРАЗОВАТЕЛЬНЫЕ ТЕХНОЛОГИИ

Образовательные технологии, которые обеспечивают достижение планируемых

результатов обучения согласно основной образовательной программе, представлены

таблицей.

Методы и формы организации обучения (ФОО)

ФОО

Методы

Лекции

(16=8+8)

Практические

занятия

(24=12+12)

Самост.

работа

IT-методы х x x

Работа в команде х х

Игра х

Поисковый метод х х х

Проектный метод х

Исследовательский метод х х х

Для достижения поставленных целей преподавания дисциплины реализуются

следующие средства, способы и организационные мероприятия:

изучение теоретического материала дисциплины на лекциях;

самостоятельное изучение теоретического материала дисциплины с использованием

Internet-ресурсов, информационных баз, методических разработок, специальной учебной и

научной литературы;

закрепление теоретического материала при проведении практических занятий,

выполнения индивидуальных заданий.

проведение лекций и практических занятий в интерактивной форме:

Лекции

№ Тема лекций Кол-во

часов

Формы и методы

1 «Тригонометрическая форма

комплексного числа».

2 IT-методы

2 «Вычисление определителей n-го

порядка. Теорема Лапласа».

2 Поисковый метод

3 «Линейные операторы». 2 IT-методы

4 «Квадрики в евклидовом

пространстве»

2 Исследовательский метод

Практические занятия

№ Тема занятий Кол-во

часов

Формы и методы

1 «Комплексные числа». 2 Работа по группам

2 «Вычисление определителей».

2 Исследовательский метод

3 «Системы линейных уравнений». 2 Работа по группам

4 «Линейные операторы» 2 Применение компьютерных

программ для расчета

собственных векторов

5 «Евклидовы пространства». 2 Подготовка презентаций по

Page 13: irbis.amursu.ru · 4 1. J : ; H Q :Я I J H = J : F F : ГЕОМЕТРИЯ И АЛГЕБРА Направление подготовки 010500.62 «Прикладная математика

13

теме «Решение задач

многомерной геометрии»

6 «Скалярное, векторное и смешанное

произведение векторов».

2 Работа по группам

7 «Геометрические свойства эллипса,

гиперболы и параболы».

2 Исследовательский метод

8 «Плоскость». 2 Работа по группам

9 «Построение поверхностей 2

порядка»

2 Применение компьютерных

программ для построения

поверхностей

10 «Приведение квадратичных форм к

каноническому виду»

2 Работа по разноуровневым

группам

11 «Приведение квадрики к

каноническому виду в евклидовом

пространстве».

2 Подготовка презентаций по

теме «Приведение квадрик

к каноническому виду»

12 «Многочлены над числовыми

полями»

2 Поисковый метод

1.8.ОЦЕНОЧНЫЕ СРЕДСТВА ДЛЯ ТЕКУЩЕГО КОНТРОЛЯ УСПЕВАЕМОСТИ,

ПРОМЕЖУТОЧНОЙ АТТЕСТАЦИИ ПО ИТОГАМ ОСВОЕНИЯ ДИСЦИПЛИНЫ

И УЧЕБНО-МЕТОДИЧЕСКОЕ ОБЕСПЕЧЕНИЕ САМОСТОЯТЕЛЬНОЙ РАБОТЫ

СТУДЕНТОВ

Теоретические вопросы к экзаменам

1 семестр

Алгебра

1. Понятие БАО и ее свойства.

2. Группа: примеры, свойства. Изоморфизм и гомоморфизм групп.

3. Кольцо: примеры, свойства. Изоморфизм и гомоморфизм колец.

4. Поле: примеры, свойства. Поле рациональных и действительных чисел.

5. Построение поля комплексных чисел.

6. Действия над комплексными числами в алгебраической форме.

7. Действия над комплексными числами в тригонометрической форме.

8. Геометрическое истолкование действий над комплексными числам.

9. Подстановки и перестановки, их свойства. Симметрическая группа.

10. Разложение подстановок в произведение циклов. Четность подстановок.

Знакопеременная группа.

11. Понятие определителя n-го порядка. Определитель 2 и 3 порядков. Свойства

определителя.

12. миноры и алгебраические дополнения. Теорема Безу и теорема Вандермонда.

13. Формулы Крамера для решения систем линейных уравнений.

14. Теорема Лапласа. Вычисление треугольного определителя и определителя некоторых

блочных матриц.

15. Действия над матрицами, свойства действий.

16. Определитель суммы и произведения матриц.

17. Понятие обратной матрицы. Элементарная матрица. Вычисление обратной матрицы

методом приписывания единичной матрицы.

18. Ранг матрицы.

19. Формула вычисления обратной матрицы.

20. Теорема о базисном миноре.

Page 14: irbis.amursu.ru · 4 1. J : ; H Q :Я I J H = J : F F : ГЕОМЕТРИЯ И АЛГЕБРА Направление подготовки 010500.62 «Прикладная математика

14

21. Арифметическое n-мерное векторное пространство.

22. Задача, приводящая к решению систем линейных уравнений. Метод Гаусса.

23. Линейные комбинации векторов. Линейная зависимость и независимость.

24. Базис и ранг конечной системы векторов.

25. Равенство строчечного и столбцового рангов.

26. Критерий совместности системы линейных уравнений.

27. Однородная система линейных уравнений.

28. Линейное пространство: определение, основные свойства и примеры.

29. Ранг и базис системы векторов. Координаты вектора относительно базиса.

30. Преобразование координат вектора при изменении базиса.

31. Размерность векторного пространства. Изофорфизм линейных пространств.

32. Подпространство векторного пространства. Линейная оболочка

33. Сумма и пересечение линейных подпространств.

34. Прямая сумма линейных подпространств.

33. Линейные многообразия в линейном пространстве.

34. Фундаментальная система решений с.л.о.у.

35. Группа, подгруппа, смежные классы.

36. Нормальные делители групп. Разложение группы по подгруппе.

37. Фактор – группа. Изоморфизм и гомоморфизм групп. Теорема о гомоморфизме.

Геометрия

1. Векторы, операции над ними.

2. Коллинеарные и компланарные векторы.

3. Линейная зависимость векторов. Базис векторного пространства. Координаты вектора.

4. Скалярное произведение векторов, его свойства.

5. Аффинная, прямоугольная и полярная системы координат. Координаты точки

6. Метод координат. Простейшие задачи в координатах.

7. Деление отрезка в заданном отношении.

8. Уравнения прямой на плоскости.

9. Полярное уравнение прямой.

10. Общее уравнение прямой на плоскости.

11. Прямая в системе координат на плоскости.

12. Взаимное расположение двух прямых на плоскости.

13. Расстояние от точки до прямой на плоскости.

14. Угол между двумя прямыми на плоскости.

15. Знак многочлена CByАx

16. Пучок прямых.

17. Эллипс. Вывод канонического уравнения.

18. Эллипс. Его свойства.

19. Гипербола. Вывод канонического уравнения.

20. Гипербола. Ее свойства

21. Парабола.

22. Уравнение линии второго прядка в полярных координатах.

23. Классификация линий второго порядка.

24. Схема приведения уравнения линии второго порядка к каноническому виду.

25. Векторы в пространстве.

26. Векторное произведение

27. Смешанное произведение

2 семестр

Алгебра

1. Многочлены над областью целостности. Теорема Безу. Схема Горнера.

2. Многочлены над полем. НОД и НОК. Алгоритм Евклида.

Page 15: irbis.amursu.ru · 4 1. J : ; H Q :Я I J H = J : F F : ГЕОМЕТРИЯ И АЛГЕБРА Направление подготовки 010500.62 «Прикладная математика

15

3. Приводимые и неприводимые многочлены. Выделение кратных множителей.

4. Многочлены над полем С.

5. Уравнения3 и 4 степени.

6. Многочлены над полем R.

7. Понятие линейного оператора. Матрица л.о. Примеры.

8. Связь между матрицами л.о. в различных базисах.

9. Действия над линейными операторами. Кольцо линейных преобразований.

9. Обратное преобразование. Вырожденное и невырожденное преобразование.

10. Инвариантные подпространства и индуцированные преобразования.

11. Ранг, образ, ядро линейного преобразования.

12. Характеристический многочлен матрицы линейного преобразования.

13. Собственные векторы и собственные значения.

14. Собственные значения симметрической матрицы.

15. Приведение матрицы линейного оператора к диагональной форме.

16. Евклидово (и унитарное) пространство. Неравенство Коши-Буняковского.

17. Длина и угол. Неравенства треугольника в евклидовом (и унитарном) пространстве.

18. Ортонормированный базис. Скалярное произведение в ортонормированном базисе.

19. Существование ортонормированного базиса. Процесс ортогонализации.

20. Матрица Грама. Критерий линейной зависимости.

21. Ортогональное дополнение. Разложение вектора на ортогональную проекцию и

перпендикуляр.

22. Линейные многообразия в евклидовом (и унитарном) пространстве.

23. Расстояния в евклидовом (и унитарном) пространстве.

24. Изоморфизм евклидовых (и унитарных) пространств.

25. Ортогональные матрицы.

26. Теорема о трансформировании симметрической матрицы в диагональную

посредством ортогональной матрицы.

27. Разложения линейного оператора.

28. Билинейные формы в линейном пространстве.

29. Приведение квадратичной формы к каноническому виду. Метод Лагранжа.

30. Метод Якоби приведения квадратичной формы к каноническому виду.

31. Закон инерции квадратичных форм. Сигнатурное правило Якоби.

32. Знакоопределѐнные квадратичные формы. Критерий Сильвестра.

33. Квадратичные формы в евклидовом (и унитарном) пространстве. Приведение к

главным осям.

34. Приведенные уравнения гиперповерхности второго порядка в евклидовом

пространстве.

35. Классификация алгебраических поверхностей второго порядка в евклидовом

пространстве.

Геометрия

1. Задание плоскости точкой и направляющим подпространством. Задание плоскости

тремя точками.

2. Задание плоскости точкой и вектором нормали. Задание плоскости «в отрезках».

Параметрические уравнения плоскости.

3. Общее уравнение плоскости.

4. Условие параллельности вектора и плоскости. Расположение плоскости в системе

координат.

5. Геометрический смысл многочлена DСzВуАхzyхР ),,( .

6. Взаимное расположение двух, трех плоскостей.

7. Угол между двумя плоскостями. Расстояние от точки до плоскости.

Page 16: irbis.amursu.ru · 4 1. J : ; H Q :Я I J H = J : F F : ГЕОМЕТРИЯ И АЛГЕБРА Направление подготовки 010500.62 «Прикладная математика

16

8. Задание прямой точкой и направляющим вектором, задание прямой двумя точками в

пространстве.

9. Задание прямой двумя пересекающимися плоскостями.

10. Взаимное расположение двух прямых в пространстве.

11. Взаимное расположение прямой и плоскости.

12. Угол между прямой и плоскостью. Угол между двумя прямыми в пространстве.

13. Расстояние от точки до прямой в пространстве.

14. Расстояние между скрещивающимися прямыми.

15. Уравнения перпендикуляра к двум скрещивающимся прямым.

16. Уравнения перпендикуляра, опущенного из данной точки на данную прямую.

17. Поверхности 2-го порядка. Метод сечений.

18. Цилиндрические поверхности. Общее уравнение цилиндра.

19. Конические поверхности. Общее уравнение конуса.

20. Поверхности вращения. Сфера.

21. Эллипсоид.

22. Однополостный гиперболоид.

23. Двуполостный гиперболоид.

24. Эллиптический параболоид.

25. Гиперболический параболоид.

26. Прямолинейные образующие поверхностей 2-го порядка.

Экзаменационный билет состоит из двух теоретических вопросов (один по алгебре,

другой по геометрии 10 баллов) и четырех задач (две задачи по алгебре и две – по

геометрии 5 баллов).

Материалы для текущего контроля, итогового контроля, тестовые

материалы, индивидуальные и коллективные задания размещены в УМКД по данной

дисциплине.

1.9.УЧЕБНО-МЕТОДИЧЕСКОЕ И ИНФОРМАЦИОННОЕ ОБЕСПЕЧЕНИЕ

ДИСЦИПЛИНЫ

а) основная литература:

1. Беклемишев Д.В. Курс аналитической геометрии и линейной алгебры. М.:

ФИЗМАТЛИТ, . 2004, 2006. – 312 с.

2. Ефимов Н.В., Розендорн Э.Р. Линейная алгебра и многомерная геометрия. –

М.:ФИЗМАТЛИТ, 2005. – 464 с.

3. Ильин В.А., Позняк Э.Г. Аналитическая геометрия. М.: Физматлит, 2007. – 224с.

б) дополнительная литература:

1. Гельфанд И.М. Лекции по линейной алгебре. М.: Добросвет: Книжный дом

Университет, 2007.

2. Клетеник Д.В. Сборник задач по аналитической геометрии – М.: Наука, 1986 .

3. Кадомцев С.Б. Аналитическая геометрия и линейная алгебра. М.: ФИЗМАТЛИТ,

2001.

в) программное обеспечение и Интернет-ресурсы:

№ Наименование ресурса Краткая характеристика

1 http://www.iqlib.ru Интернет-библиотека образовательных изданий, в

которой собраны электронные учебники,

справочные и учебные пособия. Удобный поиск по

ключевым словам, отдельным темам и отраслям

Page 17: irbis.amursu.ru · 4 1. J : ; H Q :Я I J H = J : F F : ГЕОМЕТРИЯ И АЛГЕБРА Направление подготовки 010500.62 «Прикладная математика

17

знания

2 Электронная библиотека

http://www.mccme.ru/

Сайт Московского Центра Непрерывного

Математического Образованияставит своей целью

сохранение и развитие традиций математического

образования.

3 http://www.eqworld.ipmnet.ru/ Учебно-образовательная физико-математическая

библиотека, содержащая DjVu- и PDF-файлы

учебников.

4 http://www.mathnet.ru/ Общероссийский математический портал

г) методические материалы кафедры

1. Кван Н.В. «Группы. Кольца. Поля». Учебное пособие. –Благовещенск: Изд – во АмГУ,

1999.

2. Ермак Н.В., Кван Н.В. «Векторные пространства. Методы решения задач. Ч.1.».

Учебное пособие. – Изд – во АмГУ, 2000

3. Кван Н.В. Введение в алгебру. Часть 1. Учебное пособие. – Благовещенск: Изд-во

АмГУ. 2008.

4. Кван Н.В. Введение в алгебру. Часть 2. Учебное пособие. – Благовещенск: Изд-во

АмГУ. 2008.

5. Кван Н.В. Введение в алгебру. Часть 3. Учебное пособие (электр. вариант). –

Благовещенск: Изд-во АмГУ. 2009.

6. Кван Н.В. Линейные операторы. Учебное пособие (электронный вариант). –

Благовещенск: Изд-во АмГУ. 2009.

1.10.МАТЕРИАЛЬНО-ТЕХНИЧЕСКОЕ ОБЕСПЕЧЕНИЕ ДИСЦИПЛИНЫ

(МОДУЛЯ)

Освоение дисциплины производится на базе учебных аудиторий кафедры МАиМ

АмГУ главный корпус 338-а, 519, 521, 528-а. Аудитории 338-а, 519 оснащены

компьютером, видеопроектором.

1.11. РЕЙТИНГОВАЯ ОЦЕНКА ЗНАНИЙ СТУДЕНТОВ ПО ДИСЦИПЛИНЕ

1 семестр

мо

ду

ля

Вид работы

Кол-во

баллов

1 1 ИЗ «БАО. Группа» 1

1 2 КР «Комплексные числа» 1

1 3 Тест «Комплексные числа» 1

1 4 ИЗ «Комплексные числа» 1

1 5 Баллы за выполнение ДЗ по темам модуля 1

1 6 Баллы за знание теоретического материала модуля 1

ИТОГ за 1 модуль 6

2 7 ИЗ «Вычисление определителей» 1

2 8 КР «Матрицы и определители» 1

2 9 ИЗ «Матрицы» 1

2 10 ИЗ «Системы уравнений» 1

2 11 КР «Системы линейных уравнений» 1

Page 18: irbis.amursu.ru · 4 1. J : ; H Q :Я I J H = J : F F : ГЕОМЕТРИЯ И АЛГЕБРА Направление подготовки 010500.62 «Прикладная математика

18

2 12 ТС «Определители. Матрицы. Системы уравнений» 1

2 13 Баллы за выполнение ДЗ по темам модуля 1

2 14 Баллы за знание теоретического материала 1

ИТОГ за 2 модуль 8

3 15 Тест «Векторные пространства» 1

3 16 ИЗ «Векторные пространства» 1

3 17 ИЗ «Векторы на плоскости» 1

3 18 Тест «Векторы на плоскости» 1

3 19 ИЗ «Скалярное, векторное и смешанное

произведение векторов» 1

3 20 КР «Векторы» 1

3 21 Баллы за выполнение ДЗ по темам модуля 1

3 22 Баллы за знание теоретического материала модуля 1

ИТОГ за 3 модуль 8

4 23 ИЗ «Прямая на плоскости» 1

4 24 КР «Прямая» 1

4 25 Тест «Прямая на плоскости» 1

4 26 ИЗ «Линии второго порядка» 1

4 27 КР «Линии второго порядка» 1

4 28 ТС «Линии второго порядка» 1

4 29 Баллы за выполнение ДЗ по темам модуля 1

4 30 Баллы за знание теоретического материала модуля 1

ИТОГ за 4 модуль 8

Итоговый тест (зачет) 5

Экзамен 15

ИТОГ за 1 семестр 50

2 семестр

мо

ду

ля

ВИД РАБОТЫ

5 1 ИЗ «Построение плоскостей» 1

5 2 ИЗ «Прямая и плоскость в пространстве» 1

5 3 ИЗ «Поверхности второго порядка» 1

5 4 ТС «Аналитическая геометрия в пространстве» 1

5 5 ИЗ «Пересечение поверхностей» 1

5 6 Баллы за выполнение ДЗ по темам модуля 1

5 7 Баллы за знание теоретического материала модуля 1

Итог за 5 модуль 7

6 8 ИЗ «Действия с многочленами над областью

целостности» 1

6 9 ИЗ «Многочлены над полем» 1

6 10 КР «Многочлены» 1

6 11 КР «Решение уравнений 3 и 4 степени» 1

6 12 Баллы за выполнение ДЗ по темам модуля 1

6 13 Баллы за знание теоретического материала модуля 1

Итог за 6 модуль 6

Page 19: irbis.amursu.ru · 4 1. J : ; H Q :Я I J H = J : F F : ГЕОМЕТРИЯ И АЛГЕБРА Направление подготовки 010500.62 «Прикладная математика

19

7 14 ТС «Векторные пространства» остаточные знания) 1

7 15 ИЗ «Линейные операторы» 1

7 16 ИЗ «Собственные векторы и собственные значения» 1

7 17 КЛ «Векторные пространства и линейные операторы» 1

7 18 КР «Линейные операторы» 1

7 19 Баллы за выполнение ДЗ по темам модуля 1

7 20 Баллы за знание теоретического материала модуля 1

Итог за 7 модуль 7

8 21 ИЗ «Евклидовы пространства» 1

8 22 ИЗ «Задачи многомерной геометрии» 1

8 23 КР «Евклидовы пространства» 1

8 24 ИЗ «Приведение квадратичных форм к каноническому

виду методом Лагранжа» 1

8 25 ИЗ «Приведение квадратичных форм к каноническому

виду методом Якоби и методом ортогонального

преобазования»

1

8 26 ИЗ «Квадрики» 1

8 27 КЛ «Евклидовы пространства. Квадратичные формы и

квадрики» 1

8 28 Баллы за выполнение ДЗ по темам модуля 1

8 29 Баллы за знание теоретического материала модуля 1

Итог за 8 модуль 9

Итоговый тест (зачет) 6

Экзамен 15

ИТОГ за 2 семестр 50

ИТОГ ПО ДИСЦИПЛИНЕ 100

Оценки в семестре выставляются в соответствии с положением:

«отлично» - 45-50 баллов,

«хорошо» - 35-44 балла

«удовлетворительно» - 25 – 34 балла

«неудовлетворительно» - 0 – 24 баллов

Page 20: irbis.amursu.ru · 4 1. J : ; H Q :Я I J H = J : F F : ГЕОМЕТРИЯ И АЛГЕБРА Направление подготовки 010500.62 «Прикладная математика

20

2. КРАТКОЕ ИЗЛОЖЕНИЕ ПРОГРАММНОГО МАТЕРИАЛА

АНАЛИТИЧЕСКАЯ ГЕОМЕТРИЯ

Глава I. Элементы векторной алгебры.

§1. Направленный отрезок. Вектор.

План:

1.Направленный отрезок;

2.Свойства направленных отрезков;

3.Свойства векторов;

4.Операции над векторами:

a) Сложение векторов

b) Вычитание векторов

c) Умножение векторов на число

Определение: Направленным отрезком называется множество точек прямой между двумя

точками этой прямой, одна из точек которой называется началом, вторая – концом.

Определение: Два направленных отрезка, если они лежат на одной или двух параллельных

прямых, называются коллинеарными.

Два направленных отрезка называются сонаправленными, если:

1) Лежат на одной или двух параллельных прямых;

2) Их концы находятся в одной полуплоскости

относительно прямой, проходящей через их начало.

Два направленных отрезка называются

противоположнонаправленными, если:

1) Лежат на концах одной или двух параллельных прямых;

2) Их концы находятся в разных полуплоскостях

относительно прямой проходящей через их начало.

Два направленных отрезка называются равными, если:

1) Они сонаправлены;

2) Их длины равны.

Определение: Направленный отрезок, имеющий начало и конец в одной точке

(нулевую длину), называется нулевым направленным отрезком.

Определение: Свободным вектором (вектором) называется класс направленных

отрезков равных между собой, то есть сонаправленных и равных по длине.

Замечание: Для обозначения векторов используют либо малые буквы, либо вектор

обозначают одним из представителей класса направленных отрезков.

Замечание: Множество нулевых направленных отрезков – это нулевой вектор.

Замечание: Все основные свойства направленных отрезков (коллинеарность,

сонапрвленность, противоположнонаправленность) справедливы и для вектора.

Определение: два вектора называются противоположными, если:

1) Они противоположнонаправлены;

2) Их длины равны.

Определение: Вектор, длина которого равна единице, называется единичным или ортом.

Суммой двух векторов называется вектор, который можно получить:

А

В

а

Page 21: irbis.amursu.ru · 4 1. J : ; H Q :Я I J H = J : F F : ГЕОМЕТРИЯ И АЛГЕБРА Направление подготовки 010500.62 «Прикладная математика

21

1) По правилу треугольника;

2) По правилу параллелограмма.

Правило треугольника:

1) Зафиксируем точку;

2) Отложим от фиксированной точки первый вектор;

3) От конца первого вектора отложим второй;

4) Соединим начало первого вектора и конец второго.

Правило параллелограмма:

1) Зафиксируем точку;

2) Откладывает от фиксированной точки оба вектора;

3) Полученную фигуру достраиваем до параллелограмма;

4) Проводим диагональ параллелограмма из общего начала векторов – это сумма.

Свойства сложения:

1) Коммутативность:

2) Ассоциативность:

3)

4)

Правило многоугольника:

Для сложения нескольких векторов от фиксированной точки откладывают первый

вектор, каждый следующий откладывают от конца предыдущего. Вектор суммы

начинается в начале первого вектора и заканчивается в конце последнего.

Вычитание векторов.

Определение: Разностью векторов и , называется такой вектор , что

выполняется равенство:

Вывод: Для того, чтобы построить вектор разности вектор откладывают от одной

точки, результирующий вектор направлен от конца вычитаемого к концу уменьшаемого.

Замечание: Если вектор отложить от одной точки и достроить фигуру до

параллелограмма, то диагонали параллелограмма это векторы суммы и разности

исходных.

Умножение вектора на число.

Определение: Произведением вектора на действительное число R

называется вектор , такой что:

1) Длина

2) Коллинеарны, причѐм: и сонаправлены, если ; и

противоположнонаправлены, если

Свойства:

1)

2)

3)

àââà ñâàñâà )()(

àà 0

0)( àà

à â õ

àâõ

à

àð

àð

ð à 0 ð à

0

àà 1

àà 1

00

Page 22: irbis.amursu.ru · 4 1. J : ; H Q :Я I J H = J : F F : ГЕОМЕТРИЯ И АЛГЕБРА Направление подготовки 010500.62 «Прикладная математика

22

4) Дистрибутивность относительно суммы чисел:

5) Дистрибутивность относительно суммы векторов:

6) Ассоциативность:

§2 Линейная зависимость. Базис.

План:

1. Теорема о коллинеарных векторах;

2. Теорема о компланарных и не компланарных векторах;

3. Линейная зависимость векторов, еѐ свойства;

4. Базис векторного пространства. Координаты вектора.

Теорема: Если векторы и коллинеарны, причѐм , то существует и притом

единственное, действительное число такое, что

Определение: Три вектора - компланарны, причѐм и неколлинеарны, то

существует и притом единственная пара чисел R: .

Теорема (о некомпланарныхых векторах): Если некомпланарны, то для любого

вектора существует и притом единственный набор чисел : .

Определение: Пусть дана система векторов . Вектор

называется линейной комбинацией векторов системы, где

- действительные числа.

Определение: Линейная комбинация вида - называется нулевой

линейной комбинацией.

Определение: Система векторов называется линейнозависимой, если в

нулевой линейной комбинации векторов системы хотя бы один коэффициент отличен от

нуля.

Определение: Система векторов называется линейно-независимой, если в

нулевой линейной комбинации векторов системы все числовые коэффициенты равны нулю.

Свойства линейной зависимости:

1.Система векторов линейно-зависима тогда и только тогда, когда один из

векторов системы можно выразить через остальные в виде линейной комбинации.

2.Если часть системы векторов линейно-зависима, то и вся система векторов линейно-

зависима.

3.Если вся система векторов линейно-независима, то любая еѐ часть линейно-

независима.

4.Линейно-независимая система векторов не может содержать нулевого вектора.

Определение: Базисом называется упорядоченная система векторов, удовлетворяющая

свойствам: линейно-независимая система векторов; любой вектор пространства выражается

через базисные в виде линейной комбинации.

Базисом плоскости может быть два неколлинеарных вектора. Базисом трѐхмерного

пространства является три некомпланарных вектора.

ààà )(

âàâà )(

àà )()(

à â 0à

àâ

ñâà ,, à â

, âàñ

ñâà ,,

ð R ,, ñâàð

nààà ,,, 21

nnàààâ 2211

n ,,, 21

011 nnàà

nààà ,,, 21

nààà ,,, 21

02211 nnààà 021 n

nààà ,,, 21

Page 23: irbis.amursu.ru · 4 1. J : ; H Q :Я I J H = J : F F : ГЕОМЕТРИЯ И АЛГЕБРА Направление подготовки 010500.62 «Прикладная математика

23

Замечание: Размерность пространства соответствует количеству векторов в базисе.

Согласно теореме о компланарности и некомпланарности векторов имеем:

1.Если компланарен и , то существует а1, а2 такие что

2.Если трѐхмерному пространству, в котором - базис, то существует и

притом единственный набор чисел

Определение: Числовой коэффициент в разложении вектора по векторам базиса,

называются координатами вектора в данном базисе.

Замечание: Базисные векторы в собственном базисе имеют координаты из нулей и

единицы.

Базис , то

Операции над векторами в координатах.

1. Вектор суммы имеет координаты каждая, из которых есть сумма соответствующих

координат слагаемого.

2. Вектор разности имеет координаты каждая, из которых есть разность

соответствующих координат уменьшаемого и вычитаемого.

3. При умножении вектора на число, каждая координата умножается на число.

4. Координаты коллинеарных векторов пропорциональны.

§3 Скалярное произведение векторов.

План:

1.Определение, свойства из определения;

2.Общие свойства скалярного произведения. Скалярное произведение в координатах в

ортонормированном базисе;

3.Проекция вектора. Координаты вектора;

4.Физический смысл скалярного произведения векторов.

Определение: Скалярным произведением называется число равное

произведению длин векторов на cos угла между ними.

Замечание: Для того, чтобы найти угол между векторами, их откладывают от одной

точки и рассмотреть угол между лучами с общим началом.

Следствия из определения:

1.Знак скалярного произведения зависит от угла между векторами если

векторы образуют острый угол если

векторы образуют тупой угол

2.

, векторы соноправлены

, векторы противоположнонаправлены

à 1å 2å 2211 åàåàà

à 321 ,, ååå

Rààà 321 ,,

332211 åàåàåàà

21,åå )0;1(1 å )1;0(2 å

),( âà

),cos(),( âàâàâà

,0),( âà

0),cos( âà ,0),( âà 0),cos( âà

âàâà ),(

âàâà ),(

âàâà ),(

Page 24: irbis.amursu.ru · 4 1. J : ; H Q :Я I J H = J : F F : ГЕОМЕТРИЯ И АЛГЕБРА Направление подготовки 010500.62 «Прикладная математика

24

3. , если

Либо один вектор нулевой

Либо векторы ортогональны

4.

5.Скалярное произведение на множестве векторов не является бинарной

алгебраической операцией

Свойства скалярного произведения:

1. Коммутативность:

2. Ассоциативность числового множителя:

3. Дистрибутивность:

Доказательство этих свойств проводится по определению.

Среди бесконечного числа базисов плоскости особо выделяют ортонормированный базис.

Определение: Ортонормированным базисом плоскости называется базис , такой что:

Базисные векторы единичны – орты

Базисные векторы ортогональны

С учѐтом определения любой вектор в ортонормированном базисе можно расписать

Теорема: Скалярное произведение равно сумме произведений соответствующих

координат.

Следствия из теоремы:

1.

2.

3.

Пусть дан ортонормированный базис

Рассмотрим скалярное произведение вектора с каждым из базисных векторов

0),( âà

2),( ààààà

),( ààà

),(),( àââà

),(),( âàâà

),(),(),( ñàâàñâà

ji ,

1 ji

jaiàà 21

2211),( âàâàâà

âà 02211 âàâà2

2

2

1),( àààà ààà2

2

2

1

âà

âàâà

),(),cos(

ââàà

âàâàâà

2

2

2

1

2

2

2

1

2211),cos(

ji ,

),( 21 ààà

jaiàà 21

à

112121 ),(),(),(),(),( aiiajiaiiaijaiaià

),cos(),( iaiaia

Page 25: irbis.amursu.ru · 4 1. J : ; H Q :Я I J H = J : F F : ГЕОМЕТРИЯ И АЛГЕБРА Направление подготовки 010500.62 «Прикладная математика

25

В ортонормированном базисе каждая координата это ортогональная проекция

данного вектора на соответствующий базисный вектор.

Замечание: Косинусы углов, которые образуют данный вектор с базисными,

называются направляющими косинусами данного вектора, причѐм так как

Проекция вектора на вектор:

В результате вычисления проекции вектора на вектор может быть отрицательной –

это возможно, если скалярное произведение между векторами отрицательное, а значит

угол – тупой.

Физический смысл скалярного произведения

Векторам, изображающим силу, скорость момента силы и так далее приписывается

размерность – одночлен, составленный из какого-то набора символов. Такие одночлены

перемножаются и делятся обычным способом.

§4 Система координат. Простейшие задачи. Полярные координаты

План:

I.Понятие системы координат. Координаты точки;

II.Простейшие задачи:

a. Координаты вектора;

b. Расстояние между точками;

c. Деление отрезков в заданном отношении;

d. Площадь треугольника;

III.Полярная система координат. Связь между полярными и декартовыми

координатами.

Пункт 1:

Выберем на плоскости точку О рассмотрим на плоскости базис

Отложим базисные векторы от фиксированной точки, полученная конструкция

называется системой координат 21,, ååÎ

),cos(1 iaaa

),(cos),(cos 22222

2

2

1

2jaaiaàààà

1),(cos),(cos 22 jaia

â

âààÏð â

),(

21,åå

О

1å2å

О 1å

Page 26: irbis.amursu.ru · 4 1. J : ; H Q :Я I J H = J : F F : ГЕОМЕТРИЯ И АЛГЕБРА Направление подготовки 010500.62 «Прикладная математика

26

В системе координат выделяют:

1. Точку О – начало координат

2. Координатные оси и , проходящие через начало координат в направлении

базисных векторов, причѐм длины базисных векторов – это длины мерных отрезков

по этим осям.

Выделяют различные системы координат:

1. Аффинная система координат (два некомпланарных вектора)

2. Прямоугольная система координат

3. Ортонормированная система координат

Ортонормированной системой координат принято считать систему с правым базисом

(движение от к против часовой стрелки)

Замечание: В аффинном пространстве (пространстве точек) аналогичную процедуру

задания системы координат можно осуществить вводя три упорядоченные точки.

Определение: Координатами точки М в данной системе координат, называются

координаты еѐ радиус-вектора.

Таким образом между множеством точек и множеством упорядоченных пар чисел (их

координат) установлено взаимнооднозначное соответствие.

Выберем точку О – полюс, рассмотрим луч с началом в точке О, вдоль луча выберем

единичный вектор , луч – полярная ось. Каждой точке плоскости поставим в соответствие

две характеристики:

1. Длина еѐ радиус-вектора

2. Угол между радиус-вектором и

Замечание: Для построения точки в полярных координатах работу удобно начинать со

второй координаты.

Связь между полярной и прямоугольной системой координат.

1å 2å

i j

i

i

полюс

Радиус-вектор

Полярная ось

М

i

Х

У

М

i

j

Page 27: irbis.amursu.ru · 4 1. J : ; H Q :Я I J H = J : F F : ГЕОМЕТРИЯ И АЛГЕБРА Направление подготовки 010500.62 «Прикладная математика

27

Выберем прямоугольную систему координат . Рассмотрим полярную систему

координат.

О – полюс

Ох – полярная ось

- единичный вектор

Пусть - в прямоугольной системе координат, - в полярной системе

координат.

По теореме Пифагора

, ,

§5 Метод координат на плоскости.

План:

1. Суть метода;

2. Уравнения фигуры;

3. Схема составления аналитического условия для геометрической фигуры.

Введение на плоскости системы координат установило взаимнооднозначное

соответствие между точками плоскости и упорядоченными парами действительных чисел.

Аналитическая геометрия одним из основных своих методов использует метод

координат, его суть: по средствам координат точек геометрические объекты задаются

аналитически с помощью чисел, уравнений и их систем (систем неравенств).

Достоинства метода координат: при доказательстве теорем или решений задач,

использование аналитических методов (метод координат) существенно упрощает

рассуждения и позволяет алгоритмизировать рассуждение.

Геометрическим методом точек (ГМТ) или фигурой на плоскости будем называть

множество точек, удовлетворяющих некоторому свойству.

Пример: ГМТ, удалѐнных от данной точки на данное расстояние – окружность с

центром в данной точке, данного радиуса.

Определение: Условием, определяющим фигуру в данной системе координат

называется уравнение, неравенство или их системы такие, что координаты любой точки,

принадлежащей фигуре удовлетворяют им, а координаты любой точки, не

принадлежащей этой фигуре, не удовлетворяют.

Замечание: Если условие, определяющее фигуру является уравнением , его

называют уравнением фигуры.

Линии на плоскости

Пусть х и у – две произвольные переменные.

Определение: Соотношение вида F(x,y)=0 называется уравнением, если оно

справедливо не для всяких пар чисел х и у.

jiÎ ,,

i

),( óõÌ ),( Ì

22 óõ

ySin

xCos

x

yTg

Page 28: irbis.amursu.ru · 4 1. J : ; H Q :Я I J H = J : F F : ГЕОМЕТРИЯ И АЛГЕБРА Направление подготовки 010500.62 «Прикладная математика

28

Пример: 2х + 7у – 1 = 0 , х2

+ y2

– 25 = 0.

Если равенство F(x,y)=0 выполняется для любых х, у, то, следовательно, F(x,y) = 0

– тождество.

Пример: (х + у)2

- х2

- 2ху - у2

= 0

Говорят, что числа х0 и у0 удовлетворяют уравнению, если при их подстановке в это

уравнение оно обращается в верное равенство.

Важнейшим понятием аналитической геометрии является понятие уравнения линии.

Определение: Уравнением данной линии называется уравнение F(x,y)=0, которому

удовлетворяют координаты всех точек, лежащих на этой линии, и не удовлетворяют

координаты никакой из точек, не лежащих на этой линии.

Линия, определяемая уравнением y = f(x), называется графиком функции f(x).

Переменные х и у – называются текущими координатами, т. к. являются координатами

переменной точки.

Определение: Линия, определяемая в декартовой системе координат алгебраическим

уравнением n-ой степени, называется линией n-ого порядка.

Теорема: Если в декартовой системе координат линия определяется уравнением n-ой

степени, то в другой декартовой системе координат она также определяется уравнением n-

ой степени, т. е. порядок линии не зависит от выбора системы координат.

Линии первого порядка. Прямые на плоскости.

Пусть задана декартова система координат и некоторая прямая; α – угол наклона

прямой к оси Ох.

Определение: Тангенс угла α наклона прямой к оси Ох называется угловым

коэффициентом этой прямой: k = tgα.

α O

у

α

х х2-х1

у2-у1 М1

М2

Page 29: irbis.amursu.ru · 4 1. J : ; H Q :Я I J H = J : F F : ГЕОМЕТРИЯ И АЛГЕБРА Направление подготовки 010500.62 «Прикладная математика

29

= (1)

Формула (1) – определение углового коэффициента по известным двум точкам

прямой.

Пусть прямая, неперпендикулярная оси Ох, имеет угловой коэффициент k и отсекает

на оси Оу отрезок ОВ= b.

1). Пусть М(х,у) – точка с переменными координатами (переменная точка).

Рассмотрим точку В(0, b). По формуле (1) угловой коэффициент равен

(2)

Преобразуем формулу (2) к виду

или (3)

Если точка М не принадлежит данной прямой, то уравнение (3) не выполнится,

следовательно, (3) - это уравнение прямой (по определению).

Уравнение (3) определяет прямую, имеющую угловой коэффициент k, и

отсекающую на оси Oу отрезок b, и называется уравнением прямой с угловым

коэффициентом.

2). Пусть известна одна точка М1(х1,у1) и угловой коэффициент k:

По формуле (1), если М (х, у) – переменная точка,

.

Уравнение (5) – это уравнение прямой с угловым коэффициентом k, проходящей

через точку М1 .

1) Пусть известны точки М1 (х1, у1) и М2(х2, у2), принадлежащие прямой. Найти

уравнение этой прямой.

tg kxx

yy

12

12

x

byk

kxby bkxy

kxx

yy

1

1 11 xxkyy

11, yx

b

x x

y

y-b

M

B

α

α

O

Page 30: irbis.amursu.ru · 4 1. J : ; H Q :Я I J H = J : F F : ГЕОМЕТРИЯ И АЛГЕБРА Направление подготовки 010500.62 «Прикладная математика

30

По формуле (1) – . Отсюда, с учетом (5), получим

=

или, поделив обе части равенства на ,

. (6)

Уравнение (6) – это уравнение прямой, проходящей через две заданные точки.

Угол между прямыми

Пусть существуют две прямые, неперпендикулярные оси Ох; k1 и k2 – угловые

коэффициенты этих прямых. Пусть φ – угол между заданными прямыми.

α1 – угол наклона первой прямой к оси Ох;

α2 – угол наклона второй прямой к оси

Ох.

.

tg φ = tg = , т. к. и =>

(6)

При = (прямая перпендикулярна оси Ох) формула (6) теряет смысл.

1. Условие параллельности двух прямых. Обозначим две прямые и .

| | .

2. Условие перпендикулярности двух прямых:

.

kxx

yy

12

12

1yy 1

12

12 x-xxx

yy

12 yy

12

1

12

1

yy

yy

xx

xx

12

12 21

12

tgtg1

tgtg

11tg k 22tg k

21

12

1tg

kk

kk

2

1l 2l

1l 212 kkl

1l

22l

1

2

1

kk

x

y

1

2

α1 α2

φ

0

α1

Page 31: irbis.amursu.ru · 4 1. J : ; H Q :Я I J H = J : F F : ГЕОМЕТРИЯ И АЛГЕБРА Направление подготовки 010500.62 «Прикладная математика

31

Общее уравнение прямой

Теорема. В декартовой системе координат каждая прямая определяется уравнением

первой степени и обратно, каждое уравнение первой степени определяет прямую.

Доказательство. Пусть существует прямая . Если не перпендикулярна оси Ох, то она

определяется уравнением первой степени y = kx + b. Если прямая перпендикулярна оси Ох,

то для всех точек, принадлежащих прямой, выполняется равенство x = a, что также является

уравнением первой степени.

Обратно. Пусть дано уравнение

Ax + By + C = 0. (7)

а). Если B≠0, то можно записать или .

Обозначим и . Тогда – уравнение прямой.

б). Если В = 0, то А ≠ 0. => x = .

Пусть ≡ a => х = а – уравнение прямой, параллельной оси Oу.

Уравнение Ах + Ву + С = 0 называется общим уравнением прямой, поскольку

определяет все виды прямых без исключения.

Неполное уравнение первой степени

Рассмотрим три случая, когда уравнение является неполным.

1. С = 0 => Ах + Ву = 0 – прямая проходит через начало координат.

2. В = 0 (А ≠ 0) => Ах+ С = 0 – прямая параллельна оси Оу.

3. А = 0 (В ≠ 0) => Ву + С = 0 – прямая параллельна оси Ох.

Уравнение прямой ―в отрезках‖

Пусть дано уравнение Ах + Ву + С = 0, где А ≠ 0, В ≠ 0, С ≠ 0.

Преобразуем его к виду Ах + Ву = -С и разделим на (-С):

или .

Обозначим а ≡ , b ≡ .

– уравнение прямой в отрезках. (8)

l l

l

0B

Cyx

B

A

B

Cx

B

Ay

kB

A b

B

C bkxy

0CAxA

С

A

С

1 уС

Вх

С

А1

В

С

у

А

С

х

A

С

В

С

1b

у

а

х

Page 32: irbis.amursu.ru · 4 1. J : ; H Q :Я I J H = J : F F : ГЕОМЕТРИЯ И АЛГЕБРА Направление подготовки 010500.62 «Прикладная математика

32

Числа a и b в уравнении (8) имеют геометрический смысл.

Это величины отрезков, отсекаемых прямой на

координатных осях.

Убедимся в этом. Найдем координаты точки пересечения

прямой с осью Ох:

у=0 х = а, у = 0.

Аналогично находится длина отрезка, отсекаемого прямой на оси Оу.

Совместное исследование уравнений двух прямых

Каждое уравнение определяет прямую на плоскости. Совместное решение этих

уравнений определяет общую точку этих прямых.

1). Пусть . Определитель системы , система имеет

единственное решение.

Это значит, что прямые пересекаются в одной точке. Координаты точки пересечения

находятся по формулам Крамера.

2). Пусть . Тогда возможны два случая:

а) => общих точек нет, прямые параллельны;

б) => уравнения равносильны, т.е. определяют одну и ту же прямую.

Два уравнения определяют одну прямую, если их коэффициенты

пропорциональны.

Нормаль к прямой

Пусть прямая L задана общим уравнением:

Ах + Ву + С = 0. (9)

Пусть т. М0 (х0, у0) L . =>

Ах0 + Bу0 + C = 0. (10)

1b

у

а

х

0

0

222

111

CyBxA

CyBxA

2

1

2

1

В

В

А

А 0

22

11

ВА

ВА

2

1

2

1

В

В

А

А

2

1

2

1

2

1

С

С

В

В

А

А

2

1

2

1

2

1

С

С

В

В

А

А

b

х

у

0 a

l

Page 33: irbis.amursu.ru · 4 1. J : ; H Q :Я I J H = J : F F : ГЕОМЕТРИЯ И АЛГЕБРА Направление подготовки 010500.62 «Прикладная математика

33

Вычтем (10) из (9):

А(х - х0) + В(у - у0) = 0 (11)

Выражение (11) можно рассматривать как скалярное произведение двух векторов:

и . Так как , то , и вектор является

нормалью к прямой L.

Угол между двумя прямыми

Пусть прямые L1 и L2 заданы общими уравнениями:

А1х + В1у + С1 = 0 и А2х + В2у + С2 = 0.

Нормали к прямым: и .

Угол между прямыми можно определить как угол между нормалями к этим прямым:

Тогда условие параллельности прямых – это условие коллинеарности нормалей:

.

Условие перпендикулярности прямых – это перпендикулярность нормалей:

=> А1А2 + В1В2 = 0.

Каноническое уравнение прямой

Определение. Любой вектор, отличный от нулевого, параллельный заданной прямой,

называется направляющим вектором этой прямой.

Пусть на прямой задана точка , а вектор – направляющий

вектор прямой . Точка принадлежит прямой, если вектор

параллелен вектору :

. (12)

Уравнение (12) называется каноническим уравнением прямой на плоскости.

Угол между прямыми, заданными каноническими уравнениями, определяется как

угол между направляющими векторами этих прямых:

.

Условием параллельности прямых будет условие коллинеарности их направляющих

векторов:

BAn , 000 , yyxxММ 00 MMn MMn 0 n

111 , BAn 222 , BAn

2

2

2

2

2

1

2

1

2121

21

21cosBABA

BBAA

nn

nn

2

1

2

1

B

B

A

A

021 nn

L 111 , yxM mlq ,

L yxM , 111 , yyxxMM

q

m

yy

l

xx 11

2

2

2

2

2

1

2

1

2121cosmlml

mmll

Page 34: irbis.amursu.ru · 4 1. J : ; H Q :Я I J H = J : F F : ГЕОМЕТРИЯ И АЛГЕБРА Направление подготовки 010500.62 «Прикладная математика

34

|| .

Условие перпендикулярности прямых равносильно условию равенства нулю

скалярного произведения их направляющих векторов:

.

Параметрические уравнения прямой

Пусть .

Если величины х и у рассматривать как координаты точки М при каждом значении t,

то такие уравнения называются параметрическими уравнениями траектории точки М.

Аргумент t – переменный параметр.

В каноническом уравнении прямой (12) примем одну из величин (правую или левую

часть равенства) за параметр t. Получим два уравнения

или . (13)

Уравнения (13) – это параметрические уравнения прямой на плоскости. Если

принять, что параметр t – время, то параметрические уравнения приобретают физический

смысл. Они определяют закон движения точки по прямой L.

Нормальное (нормированное) уравнение прямой

Пусть существует прямая L. Проведем вектор , перпендикулярный , через

начало координат. Р – точка пересечения прямой и нормали.

На нормали введем положительное

направление от О к Р.

Пусть - полярный угол нормали,

– полярный угол вектора . Обозначим

|ОР| = р. Выберем на прямой точку М(х,у).

Проекция вектора на нормаль определяется

как

npn = p

(14)

Найдем выражение npn через координаты точки М. Пусть – полярные

координаты точки М.

npn

=

.

1L2

1

2

12

m

m

l

lL

1L 021212 mmllL

)(

)(

ty

tx

mtyy

ltxx

1

1

mtyy

ltxx

1

1

n L

OM

L

OM

OM

OM ,

OM sinsincoscossinsincoscoscoscos

sincos yx

α θ х

у

n

l 0

P

M φ

Page 35: irbis.amursu.ru · 4 1. J : ; H Q :Я I J H = J : F F : ГЕОМЕТРИЯ И АЛГЕБРА Направление подготовки 010500.62 «Прикладная математика

35

npn = . Из (1) и (2) => или

Уравне

ние (16) – это нормальное уравнение прямой.

Расстояние от точки до прямой

Пусть М* – любая точка плоскости, d – еѐ

расстояние от данной прямой.

Определение. Отклонением точки М* от данной

прямой называется число (+d), если М* лежит по ту

сторону от прямой, куда указывает положительное

направление нормали, и (–d) – в обратном случае.

= ±d

Теорема. Пусть точка М* (х*, у*)произвольная точка плоскости, L – прямая,

заданная уравнением xcosα + ysinα – р = 0. Отклонение точки М* от этой прямой задается

формулой

. (17)

Доказательство. Проекция точки М* на нормаль – точка . Отклонение точки М* от

прямой

δ= PQ = OQ – OP.

Но OQ = npn , а ОР = р δ = npn * - р

npn = .

Таким образом, отклонение точки М* от прямой легко вычисляется, если прямая

задана нормальным уравнением. Достаточно лишь подставить в нормальное уравнение

прямой координаты точки.

Пусть прямая задана общим уравнением: Ах + Ву + С = 0, а

x cosα + y sinα – р = 0 – еѐ нормальное уравнение .

Поскольку два уравнения определяют одну прямую, их коэффициенты должны быть

пропорциональны. Уравнение

(18)

совпадает с нормальным уравнением. Тогда

; .

Отсюда можно найти :

OM sincos yx sincos yxp

0sincos pyx

pyx sin*cos*

Q

OM OM

OM sin*cos* yx pyx sin*cos*

0 CByAx

pCBA ,sin,cos 22222 sincos)( BA

х

у l

Q

M* P

0

α

p

n

Page 36: irbis.amursu.ru · 4 1. J : ; H Q :Я I J H = J : F F : ГЕОМЕТРИЯ И АЛГЕБРА Направление подготовки 010500.62 «Прикладная математика

36

– нормирующий множитель уравнения прямой.

Определим знак нормирующего множителя:

µС = - р < 0.

Следовательно, знак µ противоположен знаку С в уравнении. (Если С = 0 – знак µ

произвольный).

Уравнение пучка прямых

Определение. Совокупность всех прямых, проходящих через некоторую точку S (х0,

у0), называется пучком прямых с центром S.

Теорема. Пусть и – уравнения двух прямых,

пересекающихся в точке S ,

α и β – числа не равные нулю одновременно. Тогда

α(А1х + В1у + С1) + β(А2х + В2у + С2) = 0 (*)

– это уравнение прямой, проходящей через точку S.

Доказательство.

Докажем, что соотношение (*) является уравнением 1-ой степени.

Запишем его в виде

и покажем, что и не обращаются в ноль одновременно.

Докажем от противного: пусть . Тогда

и .

Следовательно, . Но этого не может быть, т. к. по условию прямые пересекаются.

Наше предположение оказалось неверно, и не равны нулю

одновременно, т. е. равенство (*) – уравнение с двумя переменными (х,у). Это уравнение

1-ой степени, которое определяет прямую.

Остается доказать, что эта прямая проходит через точку S. Пусть х0, у0 – координаты

точки S. Они удовлетворяют каждому из двух уравнений прямых, следовательно, для

любых значений и выполняется равенство

α(А1х0 + В1у0 + С1) + β(А2х0 + В2у0 + С2) = 0. (**)

Значит, все прямые, определяемые уравнением (*) при различных значениях и ,

проходят через точку S. Теорема доказана.

22

1

BA

0111 CyBxA 0222 CyBxA

0212121 CCyBBxAA

21 AA 21 BB

0

0

21

21

BB

AA

2

1

A

A

2

1

B

B

2

1

2

1

B

B

A

A

21 AA 21 BB

Page 37: irbis.amursu.ru · 4 1. J : ; H Q :Я I J H = J : F F : ГЕОМЕТРИЯ И АЛГЕБРА Направление подготовки 010500.62 «Прикладная математика

37

Пусть требуется найти прямую пучка (*), проходящую через заданную точку М*(х*,

у*). Должно выполняться равенство

α (А1х*+В1у*+С1) + β (А2х*+В2у*+С2) = 0.

Для любого значения можно принять . Тогда из уравнения

А1х*+В1у*+С1+ λ (А2х*+В2у*+С2) = 0 можно найти , а уравнение

А1х+В1у+С1 + λ (А2х+В2у+С2) = 0 определяет искомую прямую.

Поверхности в пространстве

Пусть – переменные.

Выражение называется уравнением, если оно выполняется не для

любых значений .

Уравнению поверхности удовлетворяют только точки поверхности и никакие

другие точки пространства.

Определение. Поверхность – это геометрическое место точек, координаты которых

удовлетворяют данному уравнению .

Пример: уравнение задает сферу с центром в

точке ( ), радиусом .

Алгебраические поверхности определяются в декартовой системе координат

алгебраическими уравнениями вида:

Уравнение – общее уравнение первой степени.

Плоскость

Теорема. В декартовой системе координат каждая плоскость определяется

уравнением первой степени. И обратно: в декартовой системе координат каждое

уравнение первой степени определяет плоскость.

Доказательство. Пусть существует произвольная плоскость . Пусть точка

. Пусть задан вектор . Пусть

– произвольная точка пространства с переменными координатами. Условие

принадлежности точки М плоскости – это перпендикулярность векторов и :

.

Выразим условие принадлежности т. к плоскости через координаты векторов

и .

Условие перпендикулярности векторов:

0

zyx ,,

0),,( zyxF

zyx ,,

0),,( zyxF

2222 )()()( rzyx

,, r

022222

0

222

LKzHyGxFyzDxyCzByAx

DCzByAx

0 DCzByAx

),,( 0000 zyxM :n

CBAnnn ,,,,0

),,( zyxM

MM 0 n

M nMM 0

M

zzyyxxMM ,, 000 CBAn ,,

Page 38: irbis.amursu.ru · 4 1. J : ; H Q :Я I J H = J : F F : ГЕОМЕТРИЯ И АЛГЕБРА Направление подготовки 010500.62 «Прикладная математика

38

(*)

– это и есть уравнение плоскости , поскольку ему удовлетворяют только точки

плоскости. Преобразуем его:

= 0 или

. (1)

Уравнение (1) – это общее уравнение плоскости. Таким образом, плоскость

действительно определяется уравнением первой степени.

Докажем второе утверждение. Пусть дано произвольное уравнение первой степени

(1): .

Пусть - решение данного уравнения .

Тогда равенство – тождество. (2)

Вычтем тождество (2) из уравнения (1), получим

. (3)

Это уравнение плоскости, проходящей через заданную точку и

имеющей нормаль (см. уравнение (*)).

Уравнение (1) равносильно уравнению (3), т. к. они получаются друг из друга путем

почленного вычитания тождества. Следовательно, уравнение (1) является уравнением той

же плоскости. Теорема доказана.

Рассмотрим два общих уравнения плоскостей. Пусть они определяют одну и ту же

плоскость (4)

Тогда нормали и коллинеарны, а, следовательно,

коэффициенты уравнений пропорциональны: .

Умножим первое уравнение системы на и вычтем его из второго уравнения.

Получим: .

Вывод. Если два уравнения определяют одну и ту же плоскость, то их коэффициенты

пропорциональны.

Неполные уравнения плоскости

Неполные уравнения получаются, когда какие-либо коэффициенты уравнения равны

нулю.

1. D=0, – плоскость проходит через начало координат.

2. A=0, , плоскость параллельна оси Ох.

3. B=0, плоскость параллельна оси .

0)()()(0 0000 zzCyyBxxAnMM

D

CzByAxCzByAx )( 000

0 DCzByAx

0 DCzByAx

000 ,, zyx

0000 DCzByAx

0)()()( 000 zzCyyBxxA

),,( 0000 zyxM

CBAn ,,

0

0

222

1111

DzCyBxA

DzCyBxA

1111 ,, CBAn 2222 ,, CBAn

121212 ,, CCBBAA

1212 0 DDDD

0 CzByAx

OxCBOnDCzBy ,,,0

Oy

Page 39: irbis.amursu.ru · 4 1. J : ; H Q :Я I J H = J : F F : ГЕОМЕТРИЯ И АЛГЕБРА Направление подготовки 010500.62 «Прикладная математика

39

0 a

b

c

X

Y

Z

4. С=0, плоскость параллельна оси .

5. A=0, B=0 плоскость параллельна плоскости .

6. A=0, C=0 плоскость параллельна плоскости .

7. B=0, C=0 плоскость параллельна плоскости .

8. A=B=D=0 Это плоскость .

9. A=C=D=0 Плоскость .

10. B=C=D=0 Плоскость .

Уравнение плоскости в «отрезках»

Пусть дано общее уравнение плоскости. Преобразуем это уравнение, разделив его на

(-D):

, .

(5)

Уравнение (5) – это уравнение плоскости «в

отрезках». Коэффициенты a, b, с определяют отрезки,

отсекаемые плоскостью на координатных осях.

Угол между плоскостями

Пусть заданы две плоскости и , имеющие

нормали соответственно

. Угол между

плоскостями определяется как угол между нормалями к

этим плоскостям.

. (6)

Условие параллельности двух плоскостей – это условие коллинеарности нормалей:

.

Условие перпендикулярности плоскостей – это перпендикулярность нормалей:

.

Oz

Oxy

Oxz

Oyz

Oxy

Oxz

Oyz

,1

,

,0

D

Cz

D

By

D

Ax

DCzByAx

DCzByAx

1

C

D

z

B

D

y

A

D

x1

c

z

b

y

a

x

1 2

22221111 ,,,,, CBAnCBAn

2

2

2

2

2

2

2

1

2

1

2

1

2121212121 coscos

CBACBA

CCBBAAnnnn

2

1

2

1

2

12121 ||||

C

C

B

B

A

Ann

021212121 CCBBAA

Page 40: irbis.amursu.ru · 4 1. J : ; H Q :Я I J H = J : F F : ГЕОМЕТРИЯ И АЛГЕБРА Направление подготовки 010500.62 «Прикладная математика

40

Уравнение плоскости, проходящей через три точки, не принадлежащие одной

прямой

Даны три точки, не лежащие на одной прямой: ,

. Пусть – произвольная точка пространства.

Точка принадлежит плоскости ( ) тогда и только тогда, когда векторы

компланарны.

Условие компланарности трех векторов – это равенство нулю их смешанного

произведения:

. (7)

Уравнение (7) – это уравнение плоскости, проходящей через три заданные точки

Нормальное уравнение плоскости. Расстояние от точки до плоскости.

Пусть существует плоскость . Проведем нормаль через начало координат

О. Пусть заданы – углы, образованные нормалью с осями координат. .

Пусть – длина отрезка нормали до пересечения с плоскостью. Считая известными

направляющие косинусы нормали , выведем уравнение плоскости .

Пусть ) – произвольная точка плоскости. Вектор единичной нормали имеет

координаты . Найдем проекцию вектора на нормаль.

.

Поскольку точка М принадлежит плоскости, то

.

(8)

Это и есть уравнение заданной плоскости, называющееся нормальным.

Расстояние от точки до плоскости

Пусть дана плоскость , М* – точка пространства, d – еѐ расстояние от

плоскости.

Определение. Отклонением точки М* от плоскости называется число (+d), если M*

лежит по ту сторону от плоскости, куда указывает положительное направление нормали

, и число (-d), если точка расположена по другую сторону плоскости:

.

Теорема. Пусть плоскость с единичной нормалью задана нормальным

уравнением:

,,, 1111 zyxM zyxM ,, 222

3333 ,, zyxM zyxM ,,

M 321 MMM

31211 ,, MMMMMM

0

131313

121212

111

zzyyxx

zzyyxx

zzyyxx

n

,, n 1n

p OP

n

zyxM ,,(

cos,cos,cosn zyxOM ,,

coscoscos zyxOMпрn

pOMпрn

0coscoscos pzyx

**,*, zyx

n

d

n

Page 41: irbis.amursu.ru · 4 1. J : ; H Q :Я I J H = J : F F : ГЕОМЕТРИЯ И АЛГЕБРА Направление подготовки 010500.62 «Прикладная математика

41

.

Пусть М* – точка пространства Отклонение т. M* от плоскости задаѐтся

выражением

. (9)

Доказательство. Проекцию т. * на нормаль обозначим Q. Отклонение точки М*

от плоскости равно

.

;

;

(9)

Правило. Чтобы найти отклонение т. M* от плоскости, нужно в нормальное

уравнение плоскости подставить координаты т. M*. Расстояние от точки до плоскости

равно .

Приведение общего уравнения плоскости к нормальному виду

Пусть одна и та же плоскость задана двумя уравнениями:

- общее уравнение,

- нормальное уравнение.

Поскольку оба уравнения задают одну плоскость, их коэффициенты

пропорциональны:

.

Первые три равенства возведем в квадрат и сложим:

.

Отсюда найдем – нормирующий множитель:

. (10)

Умножив общее уравнение плоскости на нормирующий множитель, получим

нормальное уравнение плоскости:

.

Линии в пространстве. Прямая в пространстве

В аналитической геометрии каждая линия рассматривается как пересечение двух

поверхностей и соответственно определяется заданием двух уравнений.

0coscoscos pzyx

**,*, zyx

pzyx cos*cos*cos*

M

OPOQPQ

pOMпрpOPOMпрOQ nn *,*

cos*cos*cos** zyxOMпрn

pzyx cos*cos*cos*

0 DCzByAx

0coscoscos pzyx

pDCBA ,cos,cos,cos

1coscoscos 222222 CBA

222

1

CBA

0 DCzByAx

Page 42: irbis.amursu.ru · 4 1. J : ; H Q :Я I J H = J : F F : ГЕОМЕТРИЯ И АЛГЕБРА Направление подготовки 010500.62 «Прикладная математика

42

Пусть F1(x, y, z)=0 и F2(x, y, z)=0 – уравнения двух поверхностей.

Система уравнений определяет линию, являющуюся их пересечением.

Следовательно, прямую можно задать системой двух уравнений плоскостей:

(1)

Это возможно только в том случае, когда плоскости не совпадают и не параллельны,

т.е. когда нормали 1= и 2= не коллинеарны.

Система (1) – это общее уравнение прямой в пространстве.

Через каждую прямую проходит бесконечное множество плоскостей (пучок

плоскостей).

Определение. Cовокупность всех плоскостей, проходящих через одну и ту же

прямую L, называется пучком плоскостей с центром в L.

Умножим уравнения системы (1) соответственно на коэффициенты и ,

одновременно не равные нулю.

– уравнение плоскости,

проходящей через прямую L.

(Докажите самостоятельно аналогично доказательству для пучка прямых)

Определение. Совокупность всех плоскостей, проходящих через данную точку

, называется связкой плоскостей с центром в точке M0.

Канонические уравнения прямой в пространстве

Пусть – направляющий вектор прямой L, а – точка,

принадлежащая прямой. Пусть – точка с переменными координатами.

Чтобы точка принадлежала прямой L, вектор

должен быть коллинеарным вектору :

. (2)

Уравнения (2) – это канонические уравнения прямой в пространстве. Пусть прямая L

задана общим уравнением:

Найти канонические уравнения прямой L.

0,,

0,,

2

1

zyxF

zyxF

L

.0

0

2222

1111

DzCyBxA

DzCyBxA

n 111 ,, CBA n 222 ,, CBA

022221111 DzCyBxADzCyBxA

)(0000

zyxM

nmlq ,, )(0000

zyxM

),,( zyxM

),,( zyxM

0000 ,, zzyyxxMM q

n

zz

m

yy

l

xx000

.0

0

2222

1111

DzCyBxA

DzCyBxA

Page 43: irbis.amursu.ru · 4 1. J : ; H Q :Я I J H = J : F F : ГЕОМЕТРИЯ И АЛГЕБРА Направление подготовки 010500.62 «Прикладная математика

43

1). Найдем точку . Для этого нужно задать одну из координат, а

две другие определить из решения системы (1).

2). Найдем направляющий вектор прямой .

и и .

, по определению векторного произведения

.

Зная координаты точки, принадлежащей прямой, и координаты ее направляющего

вектора, можно составить канонические уравнения прямой в виде (2).

Параметрические уравнения прямой

Получаются из канонических уравнений прямой. Пусть

.

Тогда можно записать три уравнения:

(3)

Система (3) – это параметрические уравнения прямой L, проходящей через точку

и имеющей направляющий вектор . Параметрические

уравнения прямой имеют физический смысл: они описывают движение точки вдоль

заданной прямой из начального положения , где – проекции вектора

скорости точки на координатные оси.

Уравнения прямой, проходящей через две заданные точки

Пусть заданы две точки, лежащие на прямой: и .

В качестве направляющего вектора прямой можно взять вектор :

.

Подставляем в канонические уравнения (2) координаты одной из точек, например

М1, и координаты направляющего вектора:

. (4)

Получили уравнения прямой, проходящей через две заданные точки.

LzyxM ),,( 0000

nmlq ,,

1nL 2nL q 1

n q 2

n 21 nnq

1111 ,, CBAn 2222 ,, CBAn

22

11

22

11

22

11

222

111 ,,BA

BA

CA

CA

CB

CB

CBA

CBA

kji

q

tn

zz

m

yy

l

xx

000

ntzz

mtyy

ltxx

0

0

0

),,( 0000 zyxM nmlq ,,

),,( 0000 zyxM nml ,,

),,( 1111 zyxM ),,( 2222 zyxM

21MM

121212

,, zzyyxxq

12

1

12

1

12

1

zz

zz

yy

yy

xx

xx

Page 44: irbis.amursu.ru · 4 1. J : ; H Q :Я I J H = J : F F : ГЕОМЕТРИЯ И АЛГЕБРА Направление подготовки 010500.62 «Прикладная математика

44

Угол между двумя прямыми в пространстве

Определение угла между двумя прямыми сводится к определению угла между их

направляющими векторами:

(5)

Составьте самостоятельно условие параллельности и перпендикулярности прямых в

пространстве.

Угол между прямой и плоскостью

Пусть заданы плоскость : Ax + By + Cz + D = 0

и прямая L: .

Пусть - угол между нормалью к плоскости 1= и направляющим

вектором прямой . Тогда угол между плоскостью и прямой L -

дополнительный к этому углу: . Тогда

. (6)

1). Условие параллельности прямой и плоскости: =0

2). Условие перпендикулярности прямой и плоскости:

3). Условие принадлежности прямой плоскости. Пусть – любая точка

прямой. Чтобы прямая лежала в плоскости должны выполняться два условия:

а) направляющий вектор прямой должен быть перпендикулярен нормали к

плоскости:

=0,

б) произвольная точка прямой должна лежать в плоскости:

.

Условие принадлежности двух прямых одной плоскости

Две прямые в пространстве могут пересекаться, быть параллельными или

скрещиваться. Пусть прямые заданы каноническими уравнениями:

2

2

2

2

2

2

2

1

2

1

2

1

212121

21

21cosnmlnml

nnmmll

qq

qq

n

zz

m

yy

l

xx000

n CB,A,

nmlq ,,

180

222222sincos

nmlCBA

CnBmAl

qn

qn

nq CnBmAl

nq ||

0n

C

m

B

l

A

0000 ,, zyxM

CnBmAl

0

M 0000

DCzByAx

Page 45: irbis.amursu.ru · 4 1. J : ; H Q :Я I J H = J : F F : ГЕОМЕТРИЯ И АЛГЕБРА Направление подготовки 010500.62 «Прикладная математика

45

: ; : .

Для принадлежности прямых и одной плоскости необходимо и

достаточно, чтобы три вектора , и были компланарны:

. (7)

Очевидно, что в этом случае прямые или параллельны, или пересекаются.

Для того чтобы прямые пересекались, нужно, чтобы выполнялось условие (7), а

направляющие векторы прямых не были параллельны, то есть нарушалось хотя бы одно

из равенств: .

Некоторые задачи на построение прямых и плоскостей

Задача 1. Найти уравнение плоскости , проходящей через точку

и перпендикулярной заданной прямой : .

Кратко: -? : , .

Решение.

Т.к. , то нормалью к плоскости можно считать направляющий вектор

прямой: = . Воспользуемся уравнением плоскости, проходящей через

заданную точку.

: .

Задача 2. Найти уравнение плоскости , проходящей через заданную прямую

: и точку .

Кратко. -? : , .

По условию задачи нам известны две точки, лежащие в плоскости: ,

, и вектор , параллельный плоскости. Произвольная точка

будет принадлежать плоскости, если векторы будут

компланарны:

.

1L1

1

1

1

1

1

n

zz

m

yy

l

xx

2L

2

2

2

2

2

2

n

zz

m

yy

l

xx

1L 2L

21MM

1q

2q

0

222

111

121212

nml

nml

zzyyxx

2

1

2

1

2

1

n

n

m

m

l

l

),,( 0000 zyxM

1L1

1

1

1

1

1

n

zz

m

yy

l

xx

0M 1L

1L

qn 111

,, nml

0)()()(010101 zznyymxxl

1L

1

1

1

1

1

1

n

zz

m

yy

l

xx

),,( 0000 zyxM 1L

1L 0

M

0000 ,, zyxM

1111 ,, zyxM 1111 ,, nmlq

zyxM ,, MMMMq 010 ,,

0

111

010101

000

nml

zzyyxx

zzyyxx

Page 46: irbis.amursu.ru · 4 1. J : ; H Q :Я I J H = J : F F : ГЕОМЕТРИЯ И АЛГЕБРА Направление подготовки 010500.62 «Прикладная математика

46

Это и есть уравнение искомой плоскости. Раскрыв определитель, можно получить

общее уравнение плоскости.

Задача 3. Построить плоскость , проходящую через две заданные

параллельные прямые : и : .

-?: , , .

Эта задача может решаться аналогично предыдущей. Известны две точки

и , лежащие в плоскости, и вектор ( или ),

параллельный плоскости. Произвольная точка будет принадлежать плоскости,

если выполняется условие компланарности трех векторов:

– это уравнение искомой плоскости.

Аналогично решается задача построения плоскости , проходящей через две

пересекающиеся прямые.

Задача 4. Найти уравнение прямой , проходящей через заданную точку

и перпендикулярной плоскости :

Решение. Так как прямая L перпендикулярна плоскости , то ее направляющий

вектор параллелен нормали к плоскости. Следовательно, нормаль к плоскости

может служить направляющим вектором прямой L. Запишем каноническое

уравнение прямой:

.

Задача 5. Построить прямую , перпендикулярную двум скрещивающимся

прямым.

: и : ,

и проходящую через заданную точку .

Решение. Так как прямая L перпендикулярна к прямым L1 и L2, то ее направляющий

вектор можно найти как векторное произведение направляющих векторов

этих прямых:

.

Тогда можно составить каноническое уравнение искомой прямой:

.

1L1

1

1

1

1

1

n

zz

m

yy

l

xx

2L

2

2

2

2

2

2

n

zz

m

yy

l

xx

1L 2L 21 || LL

1111 ,, zyxM 2222 ,, zyxM 1111 ,, nmlq 2q

zyxM ,,

0

111

212121

111

nml

zzyyxx

zzyyxx

L

),,( 0000 zyxM 0 DCzByAx

CBAn ,,

C

zz

B

yy

A

xx 000

L

1L1

1

1

1

1

1

n

zz

m

yy

l

xx

2L

2

2

2

2

2

2

n

zz

m

yy

l

xx

),,( 0000 zyxM

nmlq ,,

22

11

22

11

22

11

21 ;;ml

ml

nl

nl

nm

nmqqq

n

zz

m

yy

l

xx 000

Page 47: irbis.amursu.ru · 4 1. J : ; H Q :Я I J H = J : F F : ГЕОМЕТРИЯ И АЛГЕБРА Направление подготовки 010500.62 «Прикладная математика

47

Задача 6. Найти точку пересечения прямой : и плоскости

.

Решение. Чтобы найти точку пересечения прямой и плоскости, нужно решить

совместно уравнения прямой и плоскости :

Для этого приведем уравнение прямой к параметрическому виду:

(*)

и подставим выражения в уравнение плоскости.

Решим полученное уравнение с одной неизвестной t, а найденное значение подставим в

(*). Полученные значения будут координатами искомой точки пересечения.

Задача 7. Через точку провести прямую, перпендикулярную заданной

прямой : .

Решение.

1). Сначала через точку проведем плоскость , перпендикулярную

прямой :

.

2). Найдем точку пересечения прямой и плоскости : .

Для этого решим систему уравнений прямой и плоскости:

3). Через две точки и проведем прямую L:

.

Задача 8. Найти проекцию точки на плоскость :

.

Решение.

Ln

zz

m

yy

l

xx 000

: 0 DCzByAx

L

.0

000

DCzByAx

n

zz

m

yy

l

xx

ntzz

mtyy

ltxx

0

0

0

zyx ,,

zyx ,,

),,( 0000 zyxM

1L1

1

1

1

1

1

n

zz

m

yy

l

xx

),,( 0000 zyxM

1L

0)()()(:010101 zznyymxxl

1L 12222 ,, LzyxM

.0)()()( 010101

1

1

1

1

1

1

zznyymxxl

n

zz

m

yy

l

xx

),,( 0000 zyxM 2222 ,, zyxM

02

0

02

0

02

0

zz

zz

yy

yy

xx

xx

),,( 0000 zyxM

0 DCzByAx

Page 48: irbis.amursu.ru · 4 1. J : ; H Q :Я I J H = J : F F : ГЕОМЕТРИЯ И АЛГЕБРА Направление подготовки 010500.62 «Прикладная математика

48

а) Найдем прямую , проходящую через точку и перпендикулярную плоскости

:

.

б) Найдем точку пересечения прямой L и плоскости : (см. задачу 6).

Точка М1 – это искомая проекция.

Задача 9. Найти проекцию точки на прямую : .

Решение.

а) Проводим плоскость : , .

: .

б) – это искомая проекция.

Задача 10. Найти проекцию прямой : на плоскость

.

а) Через прямую проводим плоскость : . Используем условие

компланарности трех векторов: .

После преобразований получим общее уравнение плоскости

.

б) Искомая прямая задается пересечением двух плоскостей : и :

Осталось привести уравнение к каноничному виду.

Задача 11. Найти расстояние от точки до прямой .

а) Найти точку – проекцию точки на прямую .

б) Длина вектора – это искомое расстояние.

Задача 12. Найти расстояние между двумя скрещивающимися прямыми и .

а) Через прямую проводим плоскость : .

б) Находим расстояние от любой точки до .

L 0M

C

zz

B

yy

A

xxL 000:

LM1

0M 1L

1

1

1

1

1

1

n

zz

m

yy

l

xx

0

M 1L

0)()()(010101 zznyymxxl

12 LM

Ln

zz

m

yy

l

xx 000

:

0 DCzByAx

1 1L

0000 ,,,,,,,, zzyyxxMMCBAnnmlq

0: 1111 DzCyBxA

1L 1

0

0

1111 DzCyBxA

DCzByAx

0M 1L

1M 0M 1L

10MM

1L 2L

1L 1 21 || L

22 LM 1

Page 49: irbis.amursu.ru · 4 1. J : ; H Q :Я I J H = J : F F : ГЕОМЕТРИЯ И АЛГЕБРА Направление подготовки 010500.62 «Прикладная математика

49

3. МЕТОДИЧЕСКИЕ УКАЗАНИЯ ДЛЯ ПРОВЕДЕНИЯ ПРАКТИЧЕСКИХ

ЗАНЯТИЙ

1 семестр

Практическое занятие 1. Тема: «Определители 2 и 3 порядков Линейные уравнения».

План:

1. Актуализация опорных знаний.

2. Решение задач на вычисление определителей 2 и 3 порядков.

3. Решение систем линейных уравнений с 2 и 3 переменными. (Кван Н.В. Введение в

алгебру. Часть 2. Учебное пособие. – Благовещенск: Изд-во АмГУ. 2009, стр.11-19)

4.Проверочная работа:

1) Вычислить определители второго порядка:

а)76

53

; б)

sinsin

cossin ; в)

2132

3221

; г)

23 1

11

aaa

a

.

2) Вычислить определители третьего порядка:

а)

261

427

3211

; б)

342

563

284

.

3) Решить систему по формулам Крамера: а)

;15

,123

yx

yx б)

.865

,10411

yx

yx

Практическое занятие 2. Тема: «БАО и ее свойства».

План:

1. Математический диктант «Числовые множества».

2. Решение задач на определение БАО и свойства БАО. (Кван Н.В. Введение в алгебру.

Часть 1. Учебное пособие. – Благовещенск: Изд-во АмГУ. 2008, стр.3-12)

3. Проверочная индивидуальная работа:

Ва-

ри-

ант

1. Определить, являются ли

бинарными алгебраическими

операциями действия +, -,

, на указанном множестве

2. Определить, какими

свойствами обладает

указанная бинарная

операция на

множестве

действительных чисел R

3. Определить, является ли

указанное множество

группой относительно

заданной бинарной

алгебраической операции

.

1 а) ;

б)

;

2 а) ;

б)

;

Nxx :

0

bba 0\RR abba 4

Nxx :

1

2 baba 0\RR

4

abba

Page 50: irbis.amursu.ru · 4 1. J : ; H Q :Я I J H = J : F F : ГЕОМЕТРИЯ И АЛГЕБРА Направление подготовки 010500.62 «Прикладная математика

50

3 а) ;

б)

;

4 а) ;

б)

;

5 а) ;

б)

;

6 а) ;

б)

;

7 а) ;

б)

;

8 а) ;

б)

;

9 а) ;

б)

;

10 а) ;

б)

;

11 а) ;

б)

;

12 а) ;

б)

;

13 а) ;

б)

;

14 а) ;

б)

;

15 а) ;

б)

;

Практическое занятие 3. Тема: «Группы, кольца, поля».

План:

1. Актуализация опорных знаний.

2. Решение задач на определение группы, кольца, поля. (Кван Н.В. Введение в алгебру.

Часть 1. Учебное пособие. – Благовещенск: Изд-во АмГУ. 2008, стр.12-31)

3. Тестирование

ТЕСТ по теме «БАО. ГРУППА. КОЛЬЦО. ПОЛЕ»

1. Среди арифметических операций выбрать БАО на множестве 1;0;1

0\: Zxx

1,1

2 baba

2

1\R

abbaba 2

Zxx :

2,1,0

baba 2 0\RR

3

abba

Zxx :

1,0

baba 2 0\RR abba 3

Zxx :2

1,0

baba2

1

1\ R

abbaba

Zxx :12

1baba

2

1 Qbaba ,:2

baba

NnZxnx ,:

11,1,, 2 iii

abba 5 Qbaba ,:3

baba

Qxx :

0

4 aba Qbaabba ,:2

baba

0\: Qxx

1

1 baba Qbaba ,:22

baba

Qxx :

1,1

1 baba Qbaba ,:23

baba

Qxx :

1,0,1

2)( baba Qbaba ,:2

baba

Rxx :

1,0

3baba 1,1 baba

0\: Rxx

1,0 3

baba

Zbaba ,:53

baba

Rxx :

1 3

baba

Zbaba ,:222

baba

Page 51: irbis.amursu.ru · 4 1. J : ; H Q :Я I J H = J : F F : ГЕОМЕТРИЯ И АЛГЕБРА Направление подготовки 010500.62 «Прикладная математика

51

1) сложение, 2) вычитание, 3) умножение, 4) деление.

2. На каких множествах операция вычитания является БАО

1) множество натуральных чисел, 2) множество положительных действительных чисел,

3) множество рациональных чисел, 4) множество четных целых чисел.

3. Указать операции, которые являются ассоциативными на указанных множествах

1) baabba на множестве R , 2) bba на множестве R ,

3) аbba на множестве R , 4)

2

baba

на множестве R .

4. Указать операции, которые не являются коммутативными на указанных множествах

1) baba на множестве Q , 2)

22 baba на множестве R ,

3) abba на множестве R , 4)

5

abba на множестве R .

5. Указать операции на заданных множествах, которые обладают нейтральным элементом

1) bba на множестве Q , 2) 22 bаba на множестве Z ,

3) 2

baba

на множестве R , 4)

3

abba на множестве Q .

6. Указать необратимые операции на данных множествах

1) операция умножения на множестве чисел вида Zbaba ,/5 ,

2) abba на множестве Z , 3) сложение векторов плоскости,

4) умножение на множестве 0\Q .

7. Выяснить, какие множества являются группами относительно указанных операций

1) натуральные числа относительно сложения, 2) множество Q относительно умножения,

3) нечетные целые числа относительно сложения, 4) множество чисел вида 1;1

относительно умножения.

8. Указать абелевы группы

1) множество многочленов степени не выше n относительно сложения,

2) множество векторов плоскости относительно сложения,

3) множество комплексных чисел относительно умножения,

4) группа целых степеней числа 2 относительно умножения.

9. Среди данных множеств, указать те, которые являются кольцами относительно

обычных сложения и умножения

1) множество N , 2) множество всех четных чисел, 3) 0 , 4) множество чисел вида

,53ba где ba, любые целые числа.

10. Среди данных множеств, указать те, которые являются полями относительно обычных

сложения и умножения

1) множество Z , 2) множество комплексных чисел, 3) 0 , 4) множество рациональных

чисел.

.

Практическое занятие 4. Тема: «Алгебраическая форма комплексного числа».

План:

1. Актуализация опорных знаний.

2. Решение задач на действия с алгебраической формой комплексного числа. (Кван Н.В.

Введение в алгебру. Часть 1. Учебное пособие. – Благовещенск: Изд-во АмГУ. 2008,

стр.32-47)

3. Проверочная работа:

Page 52: irbis.amursu.ru · 4 1. J : ; H Q :Я I J H = J : F F : ГЕОМЕТРИЯ И АЛГЕБРА Направление подготовки 010500.62 «Прикладная математика

52

1. Выполнить действия в алгебраической форме:

а) ; б) .

2. Выполнить действия в тригонометрической форме:

а) (1+ ) ; б) ; в) .

3. Решить квадратные уравнения:

а) 22+2 +1 = 0; б)

2–(5– ) +(4– ) = 0.

4. Составить квадратное уравнение с действительными коэффициентами, имеющее одним

из корней выражение .

5. Найти значение многочлена при .

Практическое занятие 5. Тема: «Тригонометрическая форма комплексного числа».

План:

1. Актуализация опорных знаний.

2. Решение задач на действия с тригонометрической формой комплексного числа. (Кван

Н.В. Введение в алгебру. Часть 1. Учебное пособие. – Благовещенск: Изд-во АмГУ. 2008,

стр.47-57)

3. Проверочная работа: 1. Представить в тригонометрической форме числа:

1) ; 2) ; 3) 6; 4) ; 5) .

2. Решить систему:

3. Выполнить указанные действия:

1) ; 2) ; 3) . Найти .

4. Найти и изобразить на комплексной плоскости все значения следующих корней:

1) ; 2) ; 3) ; 4) ; 5) .

5. Решить квадратное уравнение: , корни уравнения записать во всех

известных формах и изобразить графически.

6. Построить множества точек, удовлетворяющих указанным соотношениям:

1) ; 2) ; 3) ; 4) .

Практическое занятие 5. Тема: «Изображение комплексных чисел».

План:

1. Математический диктант «Алгебраическая и тригонометрическая формы комплексного

числа».

2. Решение задач на изображение комплексных чисел. (Кван Н.В. Введение в алгебру.

Часть 1. Учебное пособие. – Благовещенск: Изд-во АмГУ. 2008, стр.57-66)

3. Проверочная работа: 1. Дайте геометрическую интерпретацию следующих неравенств:

а) ; б) ; в) , если .

2. Запишите с помощью неравенств следующие множества точек на комплексной

плоскости:

а) полуплоскость, расположенная строго левее мнимой оси;

i815i

ii

1

)3)(2(

3i 15 4 16 3 22 i

x x x i3 x i7

35

351

i

ix

10132765 3691220 xxxxx ix

i44 i3 i1 i43

.5)32()25(

;24)3()2(

iyixi

iyixi

3)31( i 22)1( i 55)1()( zzzf )1( if

4 1 7 1 i 5 32 i43i

i

1

2

02)21(2 iziz

12 iz3

)arg(

z )Re(1 zz 431 zz

2121 zzzz 2121 zzzz zz arg1 1z

Page 53: irbis.amursu.ru · 4 1. J : ; H Q :Я I J H = J : F F : ГЕОМЕТРИЯ И АЛГЕБРА Направление подготовки 010500.62 «Прикладная математика

53

б) первый квадрант, не включая координатных осей;

в) множество точек, отстоящих от мнимой оси на расстоянии, меньшем двух;

г) полукруг радиуса 1 (без полуокружности) с центром в точке О, расположенный не

выше действительной оси.

3. Указать на комплексной плоскости геометрическое место точек, удовлетворяющих

соотношениям:

; ; ; ; >4; 1< ; 0< <1;

; ;

>1; ; ; < ; ; ;

; ; <2; ; ;

.

Практическое занятие 7. Тема: «Перестановки, подстановки, определители».

План:

1. Актуализация опорных знаний.

2. Решение задач на действия с подстановками, вычисление определителей. (Кван Н.В.

Введение в алгебру. Часть 2. Учебное пособие. – Благовещенск: Изд-во АмГУ. 2009, стр.3-

10, стр.19-27)

3. Проверочная работа:

1. Какие отображения являются подстановками:

а) , б) , в) .

2. Перемножить подстановки в прямом и обратном порядке:

а) ;

б) .

3. Найти обратные подстановки:

а) , б) , в) .

4. Представить подстановку в виде независимых циклов и определить четность

подстановки:

а) , б) ,

в)(18)(17)(16)(15)(14)(13)(12), г) (12)(23)(34)(45)(56)(67)(78).

5. Определить число инверсий в подстановке:

15 z 22 iz 361 iz 233 iz iz 1 221 iz iz

4 iziz 3

iz

iz

zRe 5Im1 z

0Re

2Im

z

zzarg

2

4

3

2

arg4

iz

z

02

2Re

iz

iz

03

1Im

iz

iz23Re0 iz izIm

2

1Im

331

z

iz

)46Im()46Re(

Im4

iziz

ziz

)3Re(3)3Im(

Re32

iziz

ziz

2342

2431

3421

3124

32134

32431

165432

654321

653421

654321

654321

165432

163425

436251

132

321

1...432

...321 n

1...1

...21

nn

n

28635471

87654321

12345678

87654321

Page 54: irbis.amursu.ru · 4 1. J : ; H Q :Я I J H = J : F F : ГЕОМЕТРИЯ И АЛГЕБРА Направление подготовки 010500.62 «Прикладная математика

54

а) , б) , в) .

6. Вычислить определитель приведением к треугольному виду и разложением по строке

или столбцу:

, .

Практическое занятие 8. Тема: «Вычисление определителей».

План:

1. Математический диктант «Свойства определителя».

2. Решение задач на вычисление определителей. (Кван Н.В. Введение в алгебру. Часть 2.

Учебное пособие. – Благовещенск: Изд-во АмГУ. 2009, стр.28-45)

3. Проверочная работа:

1) Вычислить определители приведением к треугольному виду:

а)

5487

2354

7285

6393

; б)

1684

2361

5701

4132

.

2) Вычислить определители разложением по строке или столбцу:

а)

9343

1432

1111

1111

; б)

201042

10632

4322

2222

.

3) Применяя теорему Лапласа, вычислить определители:

а)

93211

102845

10600

67391

201100

; б)

39121811

50005

70107

30003

45948

;

2342

2431

654321

165432

12345678

87654321

.

5487

2354

7285

6393

12322

10311

31121

01220

12231

103541

27772

21161153

1310732

54321

Page 55: irbis.amursu.ru · 4 1. J : ; H Q :Я I J H = J : F F : ГЕОМЕТРИЯ И АЛГЕБРА Направление подготовки 010500.62 «Прикладная математика

55

4) Вычислить определитель

1321

121

311

321

x

nx

nx

n

C

.

Практическое занятие 9. Тема: «Решение систем линейных уравнений по формулам

Крамера».

План:

1. Актуализация опорных знаний.

2. Решение систем уравнений по формулам Крамера. (Кван Н.В. Введение в алгебру.

Часть 2. Учебное пособие. – Благовещенск: Изд-во АмГУ. 2009, стр.70-71)

3. Проверочная работа:

Решить систему уравнений по формулам Крамера:

а)

;633

,623

,62333

,4232

4321

4321

4321

4321

хххх

хххх

хххх

хххх

б)

.5234

,1223

,1322

,5432

4321

4321

4321

4321

хxхх

хххх

хххх

хххх

Практическое занятие 10. Тема: «Действия над матрицами».

План:

1. Актуализация опорных знаний.

2. Решение задач на действия с матрицами. Вычисление обратной матрицы 2-мя

способами. (Кван Н.В. Введение в алгебру. Часть 2. Учебное пособие. – Благовещенск:

Изд-во АмГУ. 2009, стр.46-59)

3. Проверочная работа:

1. Вычислите BAAB :

а)

151

423

213

,

121

211

012

BA ; б)

121

024

114

,

321

212

121

BA .

2. Найти матрицу ЕАА Т 42:

а)

101

2610

732

; б)

5012

351

184

.

3. Найти обратную матрицу методом приписывания единичной матрицы и сделать

проверку:

Page 56: irbis.amursu.ru · 4 1. J : ; H Q :Я I J H = J : F F : ГЕОМЕТРИЯ И АЛГЕБРА Направление подготовки 010500.62 «Прикладная математика

56

а)

700

030

841

; б)

713

052

001

.

Практическое занятие 11. Тема: «Обратная матрица».

План:

1. Актуализация опорных знаний.

2. Решение задач на действия с матрицами. Вычисление обратной матрицы 2-мя

способами. (Кван Н.В. Введение в алгебру. Часть 2. Учебное пособие. – Благовещенск:

Изд-во АмГУ. 2009, стр.61-70)

3. Проверочная работа: 1) Вычислить обратную матрицу 2-мя способами и сделать проверку:

а)

153

132

543

.; б)

573

452

231

.

2) Решить матричное уравнение:

а) ; б) .

Практическое занятие 12. Тема: «Решение систем уравнений методом Гаусса»

План:

1. Математический диктант.

2. Решение систем линейных уравнений методом Гаусса, исследование систем линейных

уравнений с параметрами.

3. Проверочная работа: 1) Исследовать совместность данной системы и, в случае ее совместности, найти общее

решение и одно частное решение. Сделать проверку.

а)

.42369

,33446

,24523

4321

4321

4321

xxxx

xxxx

xxxx

б)

.18311424

,7436

,5732

4321

4321

4321

xxxx

xxxx

xxxx

2)Исследовать систему с параметром и решить ее в случае совместности.

а)

;3)1(

,3)1(

,3)1(

34

23

2

aazayx

aazyax

aazуха

б)

;)1(2)1()1(

,)13(22

,1)13(2)13(

2azayаxа

azаayаx

zаауха

Практическое занятие 13. Тема «Арифметическое векторное пространство. Линейная

зависимость и независимость векторов».

План:

1. Математический диктант;

1086

442

420

271

251

320

X

8212

10113

424

304

120

415

X

Page 57: irbis.amursu.ru · 4 1. J : ; H Q :Я I J H = J : F F : ГЕОМЕТРИЯ И АЛГЕБРА Направление подготовки 010500.62 «Прикладная математика

57

I. Закончить фразу:

1. Линейно независимая система векторов коллинеарные вектора содержать

(может, не может).

2. Если система векторов содержит компланарные вектора, то система является

(какой?).

3. Линейно зависимая система векторов нулевой вектор содержать (может, не

может).

4. Линейно независимая система векторов своей частью систему линейно

зависимых векторов содержать (может, не может).

5. Если часть системы векторов линейно независима, то вся система векторов

будет (какой?).

П. Что можно сказать:

1. о системе векторов a,b,c если 2а+b-c=0;

2. о числах α и β,если для векторов а и b верны равенства. b=αа и b=βa.

3. о числах х, у, z, x1, у1, z1, если для векторов a,b,c,d верны равенства d=xa+yb+zc

и d=x1a+y1b+z1c.

2. Решение задач на свойства линейно зависимых и независимы систем векторов.

3. Определение базиса и ранга систем векторов. Вычисление ранга матрицы.

4. Проверочная работа:

1)Показать, что векторы , , образуют базис и найти координаты вектора в этом

базисе.

а) = (-3, 4 , 7), = (0, -8, 11), = (13, 1, 5), = (-19, -1, 20).

б) = (4, 0 , 9), = (10, -7, 2), = (-1, 1, 14), = (-25, 20, -11).

2)Выяснить, являются ли следующие системы векторов линейно независимыми:

а) 1 (4, 5,2,6),a

2

3

4

(2, 2,1,3),

(6, 3,3,9),

(4, 1,5,6);

a

a

a

б) 1 (1,0,0,2,5),a

2

3

4

(0,1,0,3,4),

(0,0,1,4,7),

(2, 3,4,11,12).

a

a

a

Практическое занятие 14. Тема «Исследование систем уравнений»

План

1.Решение систем тремя способами – методом Крамера, матричным методом, методом

Гаусса.

2. Исследование систем.

3. Проверочная работа:

1) Решить системы уравнений а)

.66475

,3347

,44253

4321

4321

4321

xxxx

xxxx

xxxx

б)

.0255

,1222

,132

,123

,132

321

4321

4321

4321

4321

xxx

xxxx

xxxx

xxxx

xxxx

2) Найти общее решение однородной системы линейных уравнений и фундаментальную

систему решений.

а)

.025432

,033

,0232

54321

54321

5421

xxxxx

xxxxx

xxxx

б)

.07523

,05372

,023

54321

54321

54321

xxxxx

xxxxx

xxxxx

a

b

c

d

a

b

c

d

a

b

c

d

Page 58: irbis.amursu.ru · 4 1. J : ; H Q :Я I J H = J : F F : ГЕОМЕТРИЯ И АЛГЕБРА Направление подготовки 010500.62 «Прикладная математика

58

Практическое занятие 15. Контрольная работа №1 по теме «Матрицы, определители,

системы уравнений»

Практическое занятие 16. Тема «Векторные пространства»

План

1.Решение задач на определение векторного пространства.

2. Решение задач на нахождение базиса и размерности векторных пространств.

3. Нахождение координат вектора при изменении базиса.

4. Проверочная работа:

1)Найти базисы следующих систем векторов:

а) ;65

42,

32

14,

43

21321

aaa б) ;64,3,

2

1321 iaiaa

в) .42,2,2 2

3

2

21 xfxxff

Практическое занятие №17. Тема «Фундаментальная система решений слу. Линейное

многообразие»

План

1.Математический диктант «Векторное пространство».

2. Нахождение базиса пространства решений слоу.

3. Нахождение линейного многообразия слну.

3. Проверочная работа:

1) Найти базис пространства решений

;

2) Найти линейное многообразие решений .

Практическое занятие 18. Тема «Векторы».

План

1. Актуализация опорных знаний.

2. Решение задач по теме «Векторы».

3. Проверочная работа:

1. По данным векторам а и b построить векторы:

а) bа ; б) bа ; в) аb ; г) bа 5,15,2 ; д) bа 3 ; е) bа 25,1 ; ж) bа 32

1 .

2. Дано: ABCD - параллелограмм, Е – точка пересечения диагоналей АС и BD. Найти

векторы:

а) ЕDАВ ; б) ADEB2 ; в) ЕСАВ ; г) ЕDВС .

3. Дано: ABCD - параллелограмм, О – точка пересечения диагоналей АС и BD, М –

середина ВО. Выразить СМ через АВ и АD .

4. Пусть точка О – центр правильного шестиугольника ABCDEF. Найти сумму векторов

ОЕОDОСОВОА .ОF

0327

01613114

02332

07543

4321

4321

4321

4321

xxxx

xxxx

xxxx

xxxx

6987

17543

51613114

4321

4321

4321

xxxx

xxxx

xxxx

Page 59: irbis.amursu.ru · 4 1. J : ; H Q :Я I J H = J : F F : ГЕОМЕТРИЯ И АЛГЕБРА Направление подготовки 010500.62 «Прикладная математика

59

5. Доказать, что:

а) в треугольнике АВС медианы АК, СМ и BN пересекаются в одной точке;

б) если точка Q – точка пересечения медиан треугольника АВС, то .0 QСQВQА

6. Пусть DCBA ,,, – некоторые точки плоскости или пространства, М – середина

отрезка АВ, N – середина отрезка CD, О – середина MN. Доказать, что:

а) 0 ODOСOВOА ;б) .ADBСMNMN

7. 1111 DCBABCDA - параллелепипед. Указать вектор, равный сумме:

а) CDDDСВАB 111 ; б) DCDACBDAAСBA 11 .

8. DABC – тетраэдр. Изобразить векторы:

9. Даны: 11а , 23b , 20 bа . Вычислить: .bа

10. Дано: 5а , 3b , векторы a и b образуют угол 150 0. Вычислить: bа и bа

.

Практическое занятие 19. Тема «Скалярное произведение векторов».

План

1. Математический диктант.

2. Решение задач.

3. Проверочная работа:

1. Вычислить скалярное произведение векторов а и b

, зная, что qра

23 и

qрb

4 , где р

и q

- единичные взаимно перпендикулярные векторы.

2. Зная, что векторы р

и q

, длины которых равны 26р

и 3q

, образуют угол

4

3 , найти:

а) )2

1)(2( qрqр

; б)

2)34( qр

; в) 2)( qр

.

3. Вычислить длину диагоналей параллелограмма, построенного на векторах

qра

25 и qрb

3 , если известно, что 22р

, 3q

и 4

);(

.

4. Зная, что 2р

и 5q

и 3

2);(

, определить при каком значении векторы

qpbqра

3,17 взаимно перпендикулярны.

5. Найти угол между векторами )()( cabcbар

и c

.

6. Даны точки )3;1;2( А , )2;4;1( В , )1;5;0( С . Вычислить:

а) )3)(2( ВСАВАСВА ; б) 2

АВ .

7. Даны точки )1;1;5( А , )2;4;0( В , )1;2;1( С . Вычислить внутренний угол В и

внешний угол С треугольника АВС.

8.Найти вектор р

, удовлетворяющий условиям 1,3,8 cрbрар

, где

kjiа 23 , kjib 32 , kjic 32 .

Page 60: irbis.amursu.ru · 4 1. J : ; H Q :Я I J H = J : F F : ГЕОМЕТРИЯ И АЛГЕБРА Направление подготовки 010500.62 «Прикладная математика

60

9. Найти координаты вектора х

, коллинеарного вектору а {2,1,-1}, и

удовлетворяющего условию 3ах

.

10. Найти вектор х , зная, что он перпендикулярен векторам а ={2,3,-1} и b ={1,-2,3} и

удовлетворяет условию 6)2,( kjiх .

11. Найти условие, при котором угол между векторами ххт ;5;7 и хn ;1;12

больше 900.

12. Даны три силы, приложенные к одной точке,

1;4;6,11;1;2,4;3;1 321 FFF . Вычислить работу

равнодействующей этих сил, когда точка приложения, двигаясь прямолинейно,

перемещается из положения )2;8;4( A в положение ).15;2( B

13. Найти проекции вектора 2;1;2 а на координатные оси.

14. Найти проекцию вектора 5;1;0 а на ось вектора 8;0;6 а .

Практическое занятие 20. Тема «Векторное и смешанное произведения векторов».

План

1. Актуализация опорных знаний.

2. Решение задач по теме «Векторное пространство».

3. Решение задач по тем «Смешанное произведение».

4. Проверочная работа:

1. Найти модуль векторного произведения ba , если:

а) 32а

, 3b

и 3

);(

ba

; б) 5а

, 23b

и 4

3);(

ba

.

2. Найти модуль векторного произведения ba , если:

а) 5а

, 3

1b

и 1ba

; б) 4а

, 32b

и 12ba

.

3.Найти скалярное произведение ba

, если:

а) 6а

, 33b

и 39ba ; б) 3а

, 15b

и 36ba .

4. Дано: 4а

, 3b

и 6

);(

ba

. Найти:

а) ))(( baba ; б) )43)(52( baba ; в) 2ba ; г) 2)2)(2( baba .

5. Дано: векторы a и b перпендикулярны, 10а

, 12b

. Найти:

а) ))(( baba ; б) )52)(3( baba .

6. Доказать, что а +𝑏 +𝑐 =0 тогда и только тогда, когда а , 𝑏 = 𝑏, 𝑐 = 𝑐 , а .

7. Доказать, что если а , 𝑏 = 𝑐, 𝑑 = а , 𝑐 = 𝑏 𝑑 , то векторы а − 𝑑 и 𝑏 − 𝑐 коллинеарны.

8. Установить, компланарны ли векторы 𝑎 ,𝑏 ,𝑐 :

Page 61: irbis.amursu.ru · 4 1. J : ; H Q :Я I J H = J : F F : ГЕОМЕТРИЯ И АЛГЕБРА Направление подготовки 010500.62 «Прикладная математика

61

а) 𝑎 = (-2,5,-3), 𝑏 = (1,-14,-2), 𝑐 = (1,9,-1);б) 𝑎 = (3,-2,0), 𝑏 = (2,1,7), 𝑐 = (5,-1,2).

9. Доказать, что точки A (1,2,-1), B(0,1,5), C(-1,2,1), D(2,1,3) лежат в одной плоскости.

Практическое занятие 21. Тема «Простейшие задачи в координатах. Декартова,

полярная, цилиндрическая и сферическая системы координат»

План

1. Актуализация опорных знаний.

2. Решение задач.

3. Проверочная работа:

Практическое занятие 22. Тема «Прямая линия на плоскости»

План

1.Математический диктант.

2. Решение задач.

3. Тест ТЕСТ ПО ТЕМЕ «ПРЯМАЯ ЛИНИЯ НА ПЛОСКОСТИ»

1. Какой прямой принадлежит точка А(2;-5):

1) 01932 yx ; 2) 01932 yx ;

3) 01932 yx ; 4) 01132 yx .

2. Угловой коэффициент прямой 0623 yx равен:

1) 3

2; 2)

2

3 ; 3)

2

3; 4)

3

2 .

3. Длина отрезка, отсекаемого на оси ОУ прямой 0235 ух , равна:

1) 3

2 ; 2)

2

3 ; 3) 2,5; 4) 2.

4. Среди пар прямых выбрать параллельные:

1) 071080654 ухиух ; 2)

75,00163 хуиух ;

3) 13

13 хуиху ; 4) 00 хиух .

5. Дана прямая 0154 yx . Найти угловой коэффициент перпендикулярной прямой:

1) 8,0 ; 2)

25,1 ; 3) 0,25; 4) -1,25.

6. Координаты точки А(-2;3) симметричной относительно прямой 03 yx :

1) )5;2( ; 2)

)9;2( ; 3) )3;2( ; 4) )1;6( .

7. Пряма проходящая через точки А(2;3) и В(-1;-4) имеет вид:

1) 0637 ух ; 2)

01 ух ;

3) 05 ух ; 4) 0537 ух .

8. Угол, образованный прямыми 072053 ухиух , равен:

1)135о; 2)

30

о; 3) 45

о; 4) 90

о.

9. При каком значении параметра а прямая параллельна

0583)9()2( 22 аауаха оси ординат:

1) 2 ; 2)

3 ; 3) 1 и 3

5; 4) 1.

10. При каком значении параметра а прямая проходит через начало координат

065)96( 22 аауааах оси ординат:

1) 0 ; 2) 3 ; 3) 3 ; 4) 2 и 3.

11. Уравнение прямой 0510 ух «в отрезках» имеет вид:

Page 62: irbis.amursu.ru · 4 1. J : ; H Q :Я I J H = J : F F : ГЕОМЕТРИЯ И АЛГЕБРА Направление подготовки 010500.62 «Прикладная математика

62

1)

15,05

ух; 2) 1

25

ух; 3) 510 ух ; 4) 1

5,05

ух.

12. Уравнение прямой, проходящей через точку А(8;6) и отсекающей от координатного

угла треугольник с площадью 12 кв. ед.:

1)

0123 ух ; 2) 02483 ух ; 3)

01223 ух ; 4) 023 ух .

13. Определить какое из уравнений является нормальным:

1) 0213

12

13

5 ух ; 2) 01

5

3

5

2 ух ; 3)

02 х ; 4) 02 у .

14.Расстояние между параллельными прямыми 013125026125 ухиух

равно:

1) 13; 2) 39; 3) 0; 4) 3.

15. При каком значении а прямые 01303,032 уахиухух

пересекаются в одной точке:

1) -7; 2) 7; 3) 1; 4) 0.

Практическое занятие 23. Тема «Прямя линия на плоскости»

План

1. Математический диктант.

2. Решение задач.

3. Проверочная работа:

Даны координаты вершин треугольника АВС А (-5;-1), В (5;6), С (3;-4). Найти:

а) уравнение (с угловым коэффициентом, общее, в «отрезках», нормальное, полярное)

и длину каждой стороны; б) уравнения высот; в) уравнения медиан и координаты их точки

пересечения; г) уравнения биссектрис внутренних углов треугольника и его внешнего

угла при вершине А; д) уравнения серединных перпендикуляров; е) уравнение прямой,

проходящей через вершину С, параллельно прямой АВ; ж) координаты точки пересечения

медианы АМ и высоты ВН; з) угол между медианой АМ и высотой ВН; и) координаты

центра вписанной и описанной окружности; к) радиус вписанной и описанной

окружности; л) площадь треугольника. Сделать чертеж.

Практическое занятие 24. Контрольная работа №2

Практическое занятие 25. «Эллипс»

План

1. Актуализация опорных знаний.

2. Решение задач. Построение эллипсов.

3. Проверочная работа: Построить линии:

а) ;1622 yх б) ;11636

22

yx

в) ;1259

22

yx

г) ;164 22 yх

д) ;225259 22 yх е) ;125

)5(

16

)2( 22

yx

ж) ;9)4()3( 22 yх з)

;099369189 22 yyхх и) ;026442 22 ухyх к)

.078546273 22 ухyх

2. Построить линии:

а) ;25 2ух б) ;4 2ху в) ;67 2 хху

г) ;10422

13 2 хху д) .412

4

32 2ххх

Page 63: irbis.amursu.ru · 4 1. J : ; H Q :Я I J H = J : F F : ГЕОМЕТРИЯ И АЛГЕБРА Направление подготовки 010500.62 «Прикладная математика

63

3. Определить полуоси, координаты вершин и фокусов, эксцентриситет, уравнения

директрис каждого из следующих эллипсов:

а) ;1144225

22

yx

в) ;164100

22

yx

г) ;5625259 22 yх е)

;181

)5(

144

)10( 22

yx

ж) .8100)2(9)3(25 22 yх

Практическое занятие 26. Тема «Гипербола».

План

1. Актуализация опорных знаний.

2. Решение задач по теме «Гипербола».

3. Проверочная работа:

1.Построить линии:

а) ;1916

22

yx

б) ;1369

22

yx

в) ;13625

22

yx

г) ;184

22

yx

д) ;164 22 yх е) ;225259 22 yх ж) ;19

)3(

4

)4( 22

yx

з) ;112

)5(

25

)2( 22

yx

и) ;05,64366186 22 yyхх

к) ;016484 22 ухyх л) .051546273 22 ухyх

2. Построить линии:

а) ;36 2ух б) ;42 2ху в) ;6133 2 хху

г) ;16

15342

2

33 2 уух д) .4

4

31 2хху

Практическое занятие 27. Тема «Парабола».

План

1. Актуализация опорных знаний.

2. Решение задач по теме «Парабола».

3. Проверочная работа:

1. Определить полуоси, координаты вершин и фокусов, эксцентриситет, уравнения

асимптот и директрис для каждой из гипербол:

а) ;1784441

22

yx

б) ;114425

22

yx

в) ;2516

49

9

49 22 yх е)

;181

)4(

144

)1( 22

yx

ж) .2306)2(9)3(16 22 yх

2.Построить линии:

а) ;22 ху б) ;2

12 ух в) ;42 ху г) ух 32 ;

д) );4(3

2)2( 2 yх е) );3()5( 2 yу ж) ;862 yхх

з) ;342 2 yуу и) ;05,12186 2 yхх

к) 016164 2 хyу .

Page 64: irbis.amursu.ru · 4 1. J : ; H Q :Я I J H = J : F F : ГЕОМЕТРИЯ И АЛГЕБРА Направление подготовки 010500.62 «Прикладная математика

64

3. Построить линии:

а) ;23 ух б) ;5 ух в) ;ху г) ;12

1 уу

д) ;263

11 ху е) ;2

3

15,1 хх ж) .5,0124 ху

6. Составить уравнение параболы, имеющей вершину в начале координат, зная, что:

а) парабола расположена в правой полуплоскости, симметрично относительно оси Ох и

имеет параметр 4р ;

б) парабола расположена в левой полуплоскости, симметрично относительно оси Ох и

имеет параметр 25,0р ;

в) парабола расположена в верхней полуплоскости, симметрично относительно оси Оу и

имеет параметр 5,1р ;

г) парабола расположена в нижней полуплоскости, симметрично относительно оси Оу и

имеет параметр 1р ;

д) парабола расположена симметрично относительно оси Ох и проходит через точку М (-

2; 2

1).

2 семестр

Практическое занятие 1-3. Тема «Плоскость».

План

1. Актуализация опорных знаний.

2. Построение плоскостей.

3. Решение задач по теме «Плоскость».

4. Проверочная работа.

Практическое занятие 4-6. Тема «Прямая линия в пространстве»

План

1. Актуализация опорных знаний.

2. Построение плоскостей.

3. Решение задач по теме «Прямая линия».

4. Проверочная работа.

Практическое занятие 7. Тема «Эллипсоид, гиперболоиды»

План

1. Актуализация опорных знаний.

2. Построение эллипсоидов.

3. Построение гиперболоидов – однополостного и двуполостного.

4. Проверочная работа.

Практическое занятие 8. Тема «Параболоиды»

План

1. Актуализация опорных знаний.

2. Построение параболоидов – эллиптического и гиперболического.

3. Проверочная работа.

Практическое занятие 9. Тема «Конус, цилиндр»

План

1. Актуализация опорных знаний.

2. Построение конусов и цилиндров.

3. Проверочная работа.

Практическое занятие 10. Тема «Поверхности 2 порядка».

План

Page 65: irbis.amursu.ru · 4 1. J : ; H Q :Я I J H = J : F F : ГЕОМЕТРИЯ И АЛГЕБРА Направление подготовки 010500.62 «Прикладная математика

65

1. Математический диктант.

2. Решение задач по теме «Пересечение поверхностей»

3. Тестирование

1. Координаты центра сферы 011624222 zyxzyx равны

1) (2; 1; 3); 2) (-2; 1; 3); 3) (2; -1; -3); 4) (-2; -1; -3).

2. Среди уравнений линий выбрать уравнения эллипсоида

1) 12594

222

zух

; 2) 12594

222

zух

; 3) 1999

222

zух

; 4)

9222 zyx .

3. Среди уравнений линий выбрать уравнения однополостного гиперболоида

1) 125169

222

zух

; 2) 125364

222

zух

;

3) 1222 zyx ; 4) 1753

222

zух

.

4. Среди уравнений линий выбрать уравнения эллиптического параболоида

1) zух

4126

22

; 2) yzх

468

22

; 3) zyx 22; 4) y

zх7

1625

22

.

5. Среди уравнений линий выбрать уравнения конуса

1) 0161410

222

zух

; 2) 1251625

222

zух

; 3) 03694

222

zух

; 4)

1222 zyx .

6. Среди уравнений линий выбрать уравнения гиперболического цилиндра

1) 846

22

уz

; 2) 068

22

; 3) 162 22 yx ; 4) yzх

42

2

.

7. Какую линию определяют уравнения

011432

,13412

222

zyx

zyx

1) парабола; 2) гипербола; 3) эллипс; 4) прямая.

8.Координаты вершины конуса 022516016169 222 zzyx равны

1) (0; 0; 5); 2) (5; 0; 0); 3) (0; 5; 0); 4) (0; 0; -5).

9. Уравнение 02024246 222 zyxzyx задает поверхность

1) эллипсоид; 2) однополостный гиперболоид;

3) двуполостный гиперболоид; 4) гиперболический параболоид.

Практическое занятие 11. Контрольная работа №3 по теме

«Аналитическая геометрия в пространстве»

Практическое занятие 12. Тема «Многочлены над областью целостности и полем»

План

1. Актуализация опорных знаний.

2. Решение задач по теме «Многочлены над областью целостности. Применение

схемы Горнера».

3. Решение задач по теме «Многочлены над полем».

4. Проверочная работа:

1)Найти сумму, разность и произведение многочленов:

Page 66: irbis.amursu.ru · 4 1. J : ; H Q :Я I J H = J : F F : ГЕОМЕТРИЯ И АЛГЕБРА Направление подготовки 010500.62 «Прикладная математика

66

].][[23)(,3)2(1)( xiZxxxgxxixf

2) Используя схему Горнера, найти g(a): .3,40832435)( axxxxxg

3) Используя схему Горнера, определить кратность ik корня ia многочлена f(x) и

разложить f(x) на соответствующие множители:

.12,1

,123344256627483910)( axxxxxxxxxxxf

4) Пользуясь схемой Горнера, найти значения многочлена и его производных при x = a:

.2,4102435)( axxxxf

5) Найти делитель g(x), если известны делимое f(x), частное q(x) и остаток r(x):

].[16)(,672435)(,29226323410)( xZxrxxxxqxxxxxf

6)Найти наибольший общий делитель D(x) в кольце К:

).2)(42)(83)(164()(

),164)(83)(42)(2()(];[

xxxxxq

xxxxxfxRK

7)Выделить кратные множители многочлена и найти их корни:

.252837445)( xxxxxxf

Практическое занятие 13. Тема «Решение уравнений 3 и 4 степени»

План

1. Актуализация опорных знаний.

2. Решение уравнений 3 степени по формулам Кардано.

3. Решение уравнений 4 степени методом Феррари.

4. Проверочная работа:

1. Решить уравнения:

а) 0233 ixx ; б) 0)1(463 iixx ; в) 0233 xx ; г) 0963 xx ;

д) 0883 3 xx ; е) 0134 3 xx ; ж) 06116 23 xxx .

2. Решить уравнения:

а) 013 234 xxxx ; б) 0144 234 xxxx ; в) 048204 234 xxxx ;

г) 02232 234 xxxx ; д) 01053 234 xxxx .

Практическое занятие 14. Тема «Многочлены с действительными коэффициентами,

многочлены над полем Q».

План

1. Математический диктант.

2. Решение задач.

3. Проверочная работа.

Page 67: irbis.amursu.ru · 4 1. J : ; H Q :Я I J H = J : F F : ГЕОМЕТРИЯ И АЛГЕБРА Направление подготовки 010500.62 «Прикладная математика

67

Практическое занятие 15. Тема «Линейные операторы»

План

1. Актуализация опорных знаний.

2. Решение задач на определение линейного оператора.

3. Проверочная работа:

1.Выяснить, будет ли отображение линейного вещественного пространства в себя

линейным, если:

а) xx 2)( для всякого вектора Vx ;

б) для всякого вектора 3

321 ),,( Rxxxx его образ )–,,()( 321 kxkxkxx , где

Rk – фиксированное число.

2. Известно, что

)1,0,0(1 a , )1,1,0(2 a , )1,1,1(3 a ; )5,3,2(1 b , )0,0,1(2 b , )1–,1,0(3 b —

векторы линейного пространства L, заданные своими координатами в базисе

321 ,, eee . В том же базисе найдите матрицу линейного отображения , переводящего

векторы 321 ,, aaa соответственно в векторы 321 ,, bbb .

3. Линейное отображение пространства 2R в базисе )1,2(1 a , )1,1(2 a имеет

матрицу

32

53aA ,а линейное отображение пространства 2R в базисе )2,5(1 b ,

)0,1(2 b имеет матрицу

5,15,4

5,35,7bB .Найти матрицы отображений и в

базисе 21, bb .

4. Выясните, будет ли линейным отображение пространства 3R в себя, если для любого

вектора 3

321 ),,( Rxxxx :а) ),,3()( 321 xxxx ; б) ),,–()( 23321 xxxxxx .

5. Покажите, что отображение пространства вещественных функций, определѐнных и

непрерывных на отрезке ],[ ba , ставящее в соответствие каждой функции f(x) из этого

пространства функцию x

a

dxxf )( , является линейным.

6. а) Покажите, что в трѐхмерном пространстве 3V геометрических векторов, исходящих

из начала координат O, ортогональное проектирование на некоторую плоскость,

проходящую через точку O, является линейным отображением пространства в себя.

б) Пусть — ортогональное проектирование пространства 3V на плоскость xOy

прямоугольной системы координат, а 321 ,, eee — векторы, направленные по осям

координат. Найдите матрицу в базисе 321 ,, eee и в базисе 1

'

1 ee , 2

'

2 ee ,

321

'

3 eeee , а также образ вектора 321 221 eeex двумя способами:

1) исходя непосредственно из определения ; 2) по формуле XAY , где A — матрица

в каком-либо базисе, а X и Y — столбцы координат векторов x и )(xy в том же

базисе.

Page 68: irbis.amursu.ru · 4 1. J : ; H Q :Я I J H = J : F F : ГЕОМЕТРИЯ И АЛГЕБРА Направление подготовки 010500.62 «Прикладная математика

68

Практическое занятие 16. Тема «Преобразование матрицы линейного оператора при

изменении базиса»

План

1.Математический диктант.

2. Решение задач.

Проверочная работа: Линейное отображение линейного пространства 3R имеет в

базисе )7,6–,8(1 e , )13–,7,16(–2 e , )7,3–,9(3 e матрицу

22251

20221

15181

A .

Найти матрицу B того же отображения в базисе )1,2–,1('

1 e , )2,1–,3('

2 e , )2,1,2('

3 e

.

Практическое занятие 17. Тема «Образ и ядро линейного оператора»

План

1.Математический диктант.

2. Решение задач.

Проверочная работа:

1.Линейное отображение пространства L задано матрицей

5101

1121

3110

2011

в некотором базисе 321 ,, eee 4e . Найдѐм ядро и дефект

отображения .

2. а) Найти ядро, ранг и область значений линейного отображения пространства 2M

вещественных матриц порядка 2 над полем R, если задано матрицей

1113

31374

1312

1531

в базисе

01

011e ,

00

112e ,

11

103e ,

11

004e .

б) Выясним, принадлежит ли вектор

102

422y из 2M подпространству ker .

3. Линейное отображение пространства 2M квадратных матриц порядка 2 над полем R

задано в базисе

01

011e ,

00

112e ,

11

103e ,

11

004e матрицей

1113

31374

1312

1531

Page 69: irbis.amursu.ru · 4 1. J : ; H Q :Я I J H = J : F F : ГЕОМЕТРИЯ И АЛГЕБРА Направление подготовки 010500.62 «Прикладная математика

69

Найти для вектора 4210 42 eeex

43

21 его образ 0y )( 0x и полный прообраз

вектора 0y .

Практическое занятие 18. Тема «Собственные векторы и собственные значения»

План

1.Математический диктант.

2. Решение задач.

3.Проверочная работа:

1.В линейном пространстве 3R задан базис

)0,1,1(1 a , )1,1,1(2 a , )2,2,0(3 a ,

а отображение переводит произвольный вектор 3Rx , 332211 axaxaxx в вектор

11axx . Узнайте, какие из векторов

0,2,21 b , 0,5,52 b , 2,2,23 b , 8,8,24 b , 1,2,25 b

являются собственными векторами отображения и каким собственным значениям они

отвечают.

2. Найдите собственные значения и собственные векторы линейных отображений,

заданных в некотором базисе матрицами:

а)

201

335

212

; б)

212

044

010

; в)

496

375

254

; г)

1000

0001

0000

0001

; д)

1314

3503

0011

0013

.

4. Тестирование:

1. Среди отображений пространства 3R линейными являются:

1) Ах = (5х1 - 4х2 - 3, 2х1 - х2 , х32); 2) Вх = (5х1 - 4х2 - 3х3, 2х1 - х2 , 1);

3) Сх = (5х1 - 4х2 - 3х3, 2х1 - х2 , х3) ;4) Дх = (х1 - 4х22 - 3, 2х1 - х2 ,х1- х3).

2. Образом вектора )4,2,1( x при отображении , заданного матрицей

514

510

631

A является вектор:

1) )5,3,10( y ; 2) )22,22,17( y ; 3) )22,22,17( y ; 4) )0,0,0(y

3. Прообразом вектора )5,7,4(y при отображении , заданного матрицей

014

310

102

A является вектор:

1) )4,2,1( x ; 2) )4,3,10(x ; 3) )5,7,4(x ; 4) )13,10,0( x .

4. Пусть матрица линейного оператора имеет вид

55,1

31А , а матрица оператора

имеет вид

02

14В . Тогда матрица преобразования имеет вид:

Page 70: irbis.amursu.ru · 4 1. J : ; H Q :Я I J H = J : F F : ГЕОМЕТРИЯ И АЛГЕБРА Направление подготовки 010500.62 «Прикладная математика

70

1)

62

95,5; 2)

69

25,5; 3)

5,10

110; 4)

61

10.

5. Ранг линейного оператора , заданного матрицей

516

312

104

A равен:

1) 1; 2) 0; 3) 2; 4) 3.

6. Ядром линейного отображения – ортогонального проектирования на плоскость OYZ

пространства 3R является:

1) ось OY ; 2) ось OZ ; 3) плоскость OYZ ; 4) осьOX .

7. Дефект линейного оператора, заданного матрицей

231

516

264

A равен:

1) 1; 2) 3; 3) 2; 4) 0.

8. Сумма собственных значений матрицы

200

111

201

A линейного оператора с

учетом кратности равна:

1) 1; 2) 0; 3) 4; 4) 2.

9. Вектор )1,1,0,0( x является собственным для матрицы:

1)

1314

3503

0011

0013

; 2)

3314

3303

0031

0015

; 3)

3314

3303

0031

0011

;

4)

3514

3603

0031

1117

.

10. К диагональному виду можно привести матрицы:

1)

10142

681

330

; 2)

27125

46218

36158

; 3)

022

223

356

; 4)

1111

1111

1111

1111

.

Практическое занятие 19. Тема «Евклидово пространство».

План

1. Актуализация опорных знаний.

2. Решение задач.

3. Проверочная работа:

.1) Методом ортогонализации построить ортонормированный базис подпространства ,

натянутого на следующую систему векторов пространства :

1L4R

Page 71: irbis.amursu.ru · 4 1. J : ; H Q :Я I J H = J : F F : ГЕОМЕТРИЯ И АЛГЕБРА Направление подготовки 010500.62 «Прикладная математика

71

, заданных своими

координатами в некотором ортонормированном базисе.

2). В евклидовом пространстве многочленов степени над R со скалярным

произведением, задаваемым равенством , ортогонализовать базис

3). Постройте ортонормированный базис подпространства , натянутого на следующие

системы векторов: , постройте ортогональный базис.

Практическое занятие 20-23. Тема «Ортогональное дополнение. Решение задач

многомерной геометрии».

План

1. Актуализация опорных знаний.

2. Решение задач.

1. В евклидовом пространстве подпространство L задано системой уравнений

. Найдите по одному ортогональному базису в каждом из пространств

, , .

2. Найдите ортогональную проекцию a и ортогональную составляющую b вектора v

относительно подпространства L, порожденного векторами , если:

3. Найдите в пространстве С наименьший угол между вектором и

подпространством L, порожденным вектором .

Практическое занятие 24. Тема «Метод Лагранжа и метод Якоби приведения

квадратичной формы к каноническому виду».

План

1. Актуализация опорных знаний.

2. Решение задач.

3. Проверочная работа:

1) Привести квадратичную форму к каноническому виду методом Лагранжа:

а) х12

+ 2х1х2 + 2х1х3 - 3х22 - 6х2х3 - 2х3

2 , б) 4х2

2 + х3

2 + 2х1х2 - 4х1х3 + 4х2х3,

в) 4х1х2 + 2х1х3 - 2х2х3 ..

2) Привести квадратичную форму к каноническому виду при помощи невырожденного

линейного преобразования неизвестных. Найти невырожденное преобразование,

приводящее форму к каноническому виду.

)3,2,1,0(),4,3,2,1(),1,1,1,1(),0,1,1,2(),1,0,0,1( 54321 aaaaa

2

1

0

)()(),( dxxgxfgf

.)(,)(,1)( 2

321 xxfxxfxf

04936

022

024

4321

4321

4321

xxxx

xxxx

xxxx

3R

023

032

0332

321

321

321

xxx

xxx

xxx

L L 3R

321 ,, aaa

).10,2,5,3(

),11,17,8,4(

),2,4,6,3(

),3,5,4,2(

3

2

1

v

a

a

a

31 iz

i3

Page 72: irbis.amursu.ru · 4 1. J : ; H Q :Я I J H = J : F F : ГЕОМЕТРИЯ И АЛГЕБРА Направление подготовки 010500.62 «Прикладная математика

72

а) , б)

.

3) . Привести квадратичную форму к каноническому виду методом Якоби

1. 321

23

22

21 234 xxxxx ; 2. 3121

23

22

21 224 xxxxxxx .

Практическое занятие 25. Тема «Метод ортогонального преобразования приведения

квадратичной формы к каноническому виду».

План

1. Актуализация опорных знаний.

2. Решение задач.

3. Проверочная работа:

Привести квадратичную форму к каноническому виду ортогональным преобразованием

1 . а) 322123

22

21 32244 xxxxxxx

б) 43423121 2662 xxxxxxxx

2. а) 322123

22

21

3

28

3

4xxxxxxx

б)

1342

324241312124

23

22

21

102

6662105555

xxxx

xxxxxxxxxxxxxx

Практическое занятие 26. Тема «Квадрики».

План

1. Актуализация опорных знаний.

2. Решение задач.

3. Проверочная работа

Практическое занятие 27. Контрольная работа №4 «Квадратичные формы и квадрики».

4. МЕТОДИЧЕСКИЕ УКАЗАНИЯ ПО САМОСТОЯТЕЛЬНОЙ РАБОТЕ

СТУДЕНТОВ

На самостоятельную работу студентов отводится 180 (99+81(экзамен))

часов. Она включает выполнение 4-х РГР по темам, изучаемым в семестре.

РГР№1 «Комплексные числа» 1 вариант

1. Вычислить в алгебраической форме:

.

2. Решить уравнения:

а) ; б) .

3. Данные комплексные числа записать в тригонометрической форме и

вычислить значение , записав ответ в алгебраической форме:

; ; .

4. Вычислить, записав результат в алгебраической форме:

323121

2

3

2

2

2

1 2243 xxxxxxxxx 323121

2

3

2

2

2

1 2432 xxxxxxxxx

4124

3

43

17

3

2

21 ii

i

i

i

01142 zz 0)22()5()2( 2 izizi

iz 3231 iz 12

2

2

1

z

z

Page 73: irbis.amursu.ru · 4 1. J : ; H Q :Я I J H = J : F F : ГЕОМЕТРИЯ И АЛГЕБРА Направление подготовки 010500.62 «Прикладная математика

73

.

5. Вычислить значение корня:

а) ; б) ; в) .

6. Из всех чисел z, удовлетворяющих условию , найдите такие, что выражение

принимает наименьшее значение.

7. Изобразить множество точек комплексной плоскости, удовлетворяющих условиям:

а) < < ; б) ; в) .

а) ; б) ; в) .

РГР№2 «Матрицы и определители»

I. Решить матричное уравнение. Сделать проверки обратной матрицы и решения.

II. Методом Гаусса исследовать две системы линейных уравнений на совместность, найти

их общее решение, сделать проверки, определить фундаментальную систему решений

соответствующих однородных систем.

Вариант №1

I. .II. а) ; б) .

Индивидуальное задание по теме

«Векторные пространства»

Задание 1. Показать, что векторы , , образуют базис и найти координаты вектора

в этом базисе.

1) = (-3, 4 , 7), = (0, -8, 11), = (13, 1, 5), = (-19, -1, 20).

2) = (4, 0 , 9), = (10, -7, 2), = (-1, 1, 14), = (-25, 20, -11).

3) = (-8, 13 , -7), = (-3, 1, -7), = (4, -3, 3), = (11, 0, 19).

4) = (-4, 17 , 3), = (-2, 0, 2), = (12, 6, 5), = (-20, 11, 2).

5) = (2, -3 , 14), = (7, 0, -8), = (11, 13, 0), = (-6, 7, 52).

Задание 2. Даны два базиса пространства строк: и . Найти:

а) матрицу А перехода от базиса к базису ;

б) матрицу обратного перехода;

в) координаты в обоих базисах;

г) координаты вектора в базисе , имеющего во втором базисе координаты (1, 1,

1).

1)

2)

12

2018

)3(

)322(88

i

ii

5 12 6 3i 4 232 i

25 zz

izz 77

2 izi )1( 22

3Re4

arg3

21

z

z

z

1)

21Im(

zz

6 6 3

27

1i 4

2

3

2

1i

101

110

211

211

121

112

X

1

364

5432

4321

4321

4321

xxxx

xxxx

xxxx

116231110

110892

1242

4321

4321

4321

xxxx

xxxx

xxxx

a

b

c

d

a

b

c

d

a

b

c

d

a

b

c

d

a

b

c

d

a

b

c

d

321 ,, eee 321 ,, fff

321 ,, eee 321 ,, fff

1

1e

a 321 ,, eee

)3,1,1(),0,1,2(),1,1,1();2,0,1(),1,0,1(),1,1,0( 321321 fffeee

)3,1,1(),0,1,2(),1,1,1();2,0,1(),1,2,1(),1,1,1( 321321 fffeee

Page 74: irbis.amursu.ru · 4 1. J : ; H Q :Я I J H = J : F F : ГЕОМЕТРИЯ И АЛГЕБРА Направление подготовки 010500.62 «Прикладная математика

74

3)

4)

Задание 3. Найти общее решение однородной системы линейных уравнений и

фундаментальную систему решений.

1)

2)

3)

4)

5)

Индивидуальное задание по теме «Векторные пространства. Линейные операторы»

Вариант 1. 1. Образуeт ли линейное пространство заданное множество, в котором

определены сумма любых элементов a и b и произведение любого элемента а на

произвольное число :

множество всех векторов, являющихся линейными комбинациями векторов x, y, z ;

сумма a + b; произведение a.

2. Найти координаты вектора Х в базисе (е',е

', е

'), если он задан в базисе (е, е, е):

Х = { 1, 4, 8} .

3. Пусть Х = {х1, х2, х3} . Являются ли линейными следующие преобразования:

Ах = (2х1 + х2 , x2 - 2х3, 4х1 - 5х22 - 6х3 );

Вх = (2х1 + х2 , x2 - 2х3, 3х1 - 4х2 - 5х3);

Сх = (2х1 + x2 , x2 - 2, 3х1 - 4х2 - 5) .

4. Найти матрицу линейного преобразования в базисе (е1', е2

', е3

'), где

е1 ' = а11 е1 + а12 е2 + а13 е3

е2 ' = а21 е1 + а22 е2 + а23 е3

е3 ' = а31 е1 + а32 е2 + а33 е3 ,

если она задана в базисе (е1 , е2, е3) матрицей ,

|| аij || =.

5. Доказать линейность, найти матрицу, область значений и ядро оператора

проектирования на плоскость y = 0.

Индивидуальное задание по теме «Линейные операторы»

Вариант 1.

1.Определить, является ли оператор линейным, если ,

, где .

)3,1,1(),0,1,2(),1,1,1();2,0,1(),1,1,1(),1,2,0( 321321 fffeee

)3,1,1(),0,1,2(),1,1,1();2,0,1(),0,0,1(),2,1,0( 321321 fffeee

.0472

,034563

,0232

54321

54321

54321

xxxxx

xxxxx

xxxxx

.02324

,03224

,043236

54321

54321

54321

xxxxx

xxxxx

xxxxx

.032

,02

,02432

4321

5321

54321

xxxx

xxxx

xxxxx

.032

,023

,032

4321

4321

4321

xxxx

xxxx

xxxx

.02

,0223

,0453

54321

54321

4321

xxxxx

xxxxx

xxxx

.025432

,033

,0232

54321

54321

5421

xxxxx

xxxxx

xxxx

.07523

,05372

,023

54321

54321

54321

xxxxx

xxxxx

xxxxx

.0755104

,022

,022

54321

54321

54321

xxxxx

xxxxx

xxxxx

.043333

,02

,02

54321

54321

54321

xxxxx

xxxxx

xxxxx

.0622

,0323

,0

5432

54321

54321

xxxx

xxxxx

xxxxx

VV : 3RV

321 ,, xxxx 332211 ,, kxkxkxx 3Rk

Page 75: irbis.amursu.ru · 4 1. J : ; H Q :Я I J H = J : F F : ГЕОМЕТРИЯ И АЛГЕБРА Направление подготовки 010500.62 «Прикладная математика

75

2. Оператор векторного пространства ( ) в некотором базисе

имеет матрицу . Показать, что система векторов образует базис в и найти

матрицу оператора в этом базисе, если:

3. Линейный оператор в базисе имеет матрицу . Найти матрицу

этого оператора в базисе , если:

4. Найти: 1) ранг и образ линейного оператора;

2) дефект и ядро линейного оператора

если линейный оператор в базисе задан

матрицей

.

5. Линейный оператор векторного пространства задан в некотором базисе

матрицей . Найти собственные значения и собственные векторы, если:

.

Индивидуальное задание по теме

«Квадратичная форма»

Задание 1. Привести квадратичную форму к каноническому виду при помощи

невырожденного линейного преобразования неизвестных. Найти невырожденное

преобразование, приводящее форму к каноническому виду.

1) .

2)

3)

4)

5)

Задание 2. Найти ортогональное преобразование, приводящее следующие формы к

каноническому виду (приведение к главным осям), и написать этот канонический вид.

1)

2) .

3) .

4) .

5) .

ИНДИВИДУАЛЬНОЕ ЗАДАНИЕ ПО ТЕМЕ «ВЕКТОРЫ»

V 3dim V 321 ,, eee

A 321 ,, eee V

,

023

112

133

A

.3

,2

,32

3213

3212

3211

eeee

eeee

eeee

33: RR 321 ,, eee A

B 321 ,, eee

,1,3,2

,3,2,0

,1,1,1

3

2

1

e

e

e

,

327

515

893

A

.1,1,0

,1,3,3

,2,1,2

3

2

1

e

e

e

4321 ,,, eeee

7501

410123

211174

1341

A

VV : V

A

548

234

447

A

323121

2

3

2

2

2

1 2243 xxxxxxxxx

323121

2

3

2

2

2

1 2432 xxxxxxxxx

323121

2

3

2

2

2

1 222 xxxxxxxxx

323121

2

3

2

2

2

1 2223 xxxxxxxxx

323121

2

3

2

2

2

1 2243 xxxxxxxxx

.44756 3121

2

3

2

2

2

1 xxxxxxx

323121

2

3

2

2

2

1 204162511 xxxxxxxxx

323121

2

3

2

2

2

1 2265 xxxxxxxxx

323121

2

3

2

2

2

1 444 xxxxxxxxx

323121

2

3

2

2

2

1 844141417 xxxxxxxxx

Page 76: irbis.amursu.ru · 4 1. J : ; H Q :Я I J H = J : F F : ГЕОМЕТРИЯ И АЛГЕБРА Направление подготовки 010500.62 «Прикладная математика

76

1 вариант

1. По данным векторам bа, построить каждый из следующих векторов:

а) bа 5,12 (по правилу треугольника и по правилу параллелограмма);

б) bа 5,0 ; в) ab 2 .

2. Дан правильный шестиугольник ABCDEF , точка О – центр шестиугольника. Найти

на чертеже векторы АDEB 2

1и .FDОАBD

3. В параллелепипеде 1111 DCBABCDA заданы векторы, совпадающие с его ребрами:

cAAbАDаAB 1,, . Построить каждый из следующих векторов: а)

сbа 2

1; б) сbа .

4. Дан правильный шестиугольник ABCDEF , точка О – центр шестиугольника. Приняв

за базисные векторы АBи DЕ . Найти координаты векторов AЕ , FЕ и .DF

5. Даны три вектора .2;1;3,1;1;2,3;2;1 rqp Найти разложение

вектора 3;7;7а по базису .,, rqp

6. Даны три вектора kjiа 3 , kjib 52 , kjic 254 . Найти

)4( bапрс

.

7. Вектор p противоположно направлен с вектором )6;5;4( q и 733p . Найти

сумму координат вектора p .

8. Дано: 2а ,23b оbа 135);(

. Найти )23)(2( baba .

9. Даны четыре точки )2;3(),1;1(),3;2(),0;4( DСВА . Найти скалярное

произведение ))(( CBDABDАC .

10.В четырехугольнике ABCD заданы векторы

)2;4;3(),1;8;2(),5;1;4( ADBCAВ , а векторы т и n - его диагонали. Найти

модуль скалярного произведения векторов т и n .

11.Даны векторы )1;2;3( а , )5;0;4(b и bac 32 . Найти скалярное произведение

векторов а и c .

12.Найти условие, при котором угол между векторами )2;4;6( хт и )8;6;2( хn не

меньше 900.

13.Указать градусную меру угла между вектором )32;6;4( а и осью абсцисс.

14.Даны три силы, приложенные к одной точке,

)4;10;5(),9;10;3(),4;7;6( 321 FFF . Вычислить работу равнодействующей этих

сил, когда точка приложения, двигаясь прямолинейно, перемещается из положения

)2;2;3( A в положение ).2;8;3( B

15. Дано: 3а ,4b3

);(

. Найти )32)(3( baba .

Page 77: irbis.amursu.ru · 4 1. J : ; H Q :Я I J H = J : F F : ГЕОМЕТРИЯ И АЛГЕБРА Направление подготовки 010500.62 «Прикладная математика

77

16. Сила )4;0;3(Р приложена к точке )1;2;5( А . Определить величину и

направляющие косинусы момента этой силы относительно точки )1;2;0( В .

17. На векторах АВ kji 22 , АС ji 43 , АD kji 25724

построена треугольная пирамида. Найти: а) длину его ребер; б) величину угла ВСА; в)

площадь треугольника АВС ; г) объем пирамиды ; д) высоту пирамиды, опущенную из

вершины D; е) высоту треугольника АВС , опущенную из вершины В; ж) вектор АМ , где

М – точка пересечения медиан треугольника BDC; з) направляющие косинусы вектора

АС .

ИНДИВИДУАЛЬНОЕ ЗАДАНИЕ «ЛИНИИ ВТОРОГО ПОРЯДКА»

1 вариант

1.Определить тип линии (эллипс, гипербола, парабола). Для эллипса определить

координаты вершин, координаты фокусов, эксцентриситет, уравнения директрис. Для

гиперболы определить координаты вершин, координаты фокусов, эксцентриситет,

уравнения асимптот, уравнения директрис. Для параболы определить координаты

вершины, координаты фокуса, уравнение директрисы. Построить линии.

1) ,0420854 22 ухух 2)

,043181694 22 ухух

3) ,0542 2 ухх 4)

.0463 2 уух

2. Построить линии: 1) ,25 2ху 2)

,255

2 2ух

3) ,653 2хху 4)

.632 ух

3. Изобразить область, ограниченную линиями: ),1(22 22 уухх

,272 22 хухх ,0х 0у ).1,1( ух

4. Записать уравнение окружности, проходящей через правый фокус гиперболы

36486457 22 ух и имеющей центр в точке А (2;8).

5. Написать уравнение окружностей радиуса 7R , касающихся прямой

012 ух точке ).1;3(1M

6. Установить какое геометрическое место точек задает уравнение .

cos23

12

5. КОНТРОЛЬ ЗНАНИЙ

КОНТРОЛЬНАЯ РАБОТА ПО ТЕМЕ «КОМПЛЕКСНЫЕ ЧИСЛА»

1. Вычислить: | |.

2. Решить уравнение:

248

2

2

421

21

5

1

5

2)1)(1(

3)42()42)(3(i

ii

iiii

iiii

Page 78: irbis.amursu.ru · 4 1. J : ; H Q :Я I J H = J : F F : ГЕОМЕТРИЯ И АЛГЕБРА Направление подготовки 010500.62 «Прикладная математика

78

.

3. Вычислить:

а) ; б) .

4. Изобразить на плоскости ХОУ множества точек, для которых:

а) ; б) ; в) .

КОНТРОЛЬНАЯ РАБОТА ПО ТЕМЕ «МАТРИЦЫ И ОПРЕДЕЛИТЕЛИ»

1. Исследовать систему и решить ее методом Гаусса в зависимости от значений

буквенных параметров:

2. Решить методом Гаусса:

3. Найти матрицу, обратную данной, и выполнить проверку (первым способом):

.

4. Решить матричное уравнение, выполнить проверку (обратную матрицу искать по

формуле): .

5. Решить систему линейных уравнений по правилу Крамера:

6. Вычислить методом окаймления миноров ранг матрицы :

.

КОНТРОЛЬНАЯ РАБОТА ПО ТЕМЕ «ВЕКТОРНЫЕ ПРОСТРАНСТВА»

036)1(2 iziz

24

2)

2

31(

i5

1248

16

i

22 z3

2arg

4

z 1Im0 z

.1

,4)1(

,3)1(

,1

4321

4321

4321

4321

xxxx

xxxx

xxxx

xxxx

.11177142

,1755104

,122

,122

54321

54321

54321

54321

xxxxx

xxxxx

xxxxx

xxxxx

153

174

584

А

100

010

001

1094

752

221

432

211

321

X

.1364

,0495

,0543

,84352

4321

4321

4321

4321

xxxx

xxxx

xxxx

xxxx

А

31771

17401810

71884

11040

А

Page 79: irbis.amursu.ru · 4 1. J : ; H Q :Я I J H = J : F F : ГЕОМЕТРИЯ И АЛГЕБРА Направление подготовки 010500.62 «Прикладная математика

79

1. Выяснить, линейно зависима или независима система векторов .

2. Выяснить, является ли множество векторов пространства вещественных

квадратных матриц порядка 2 над полем R, состоящее из матриц вида ,

подпространством.

3. Найти многообразие решений системы линейных уравнений

.

4. Указать любой базис пространства многочленов от одной переменной степени

меньше либо равной 4 над полем R.

Контрольная работа по теме «Линейные операторы»

1. Выясните, будет ли линейным отображение пространства в себя, если для

любого вектора :

а) ; б) .

2. Найдите матрицу, образ и ядро линейного отображения пространства в базисе

, , , если известно, что оно любой вектор

переводит в вектор

а) ; б) .

3. Найдите собственные значения и собственные векторы линейного отображения ,

заданного в базисе матрицей ,и покажите, что

подпространство, натянутое на векторы и , инвариантно

относительно .

4. Выясните, какие из следующих матриц линейных отображений можно привести к

диагональному виду путѐм перехода к новому базису. Найдите этот базис и

соответствующую ему матрицу: а) ; б) .

КОНТРОЛЬНАЯ РАБОТА по теме

«Приведение квадратичной формы к каноническому виду»

1) Приведите к каноническому виду методом Лагранжа квадратичные формы:

а) , б) , в) ,

xx cos,sin,1

ab

ba

33323325

434640439

957113

6185717617

vuzyx

vuzyx

vuzx

vuzyx

3R3

321 ),,( Rxxxx

),,3()( 321 xxxx ),,–()( 23321 xxxxxx

3R

)0,0,1(1 e )0,1,0(2 e )1,0,0(3 e

),,( 321 xxxx

)2,,()( 321 xxxx )0,,()( 2131 xxxxx

4321 ,,, aaaa

2112

1012

2410

1201

21 2aa 432 2aaa

604

404

614

1111

1111

1111

1111

2

221

2

1 хххх 21хх 2

3

2

23231

2

1 424 ххххххх

Page 80: irbis.amursu.ru · 4 1. J : ; H Q :Я I J H = J : F F : ГЕОМЕТРИЯ И АЛГЕБРА Направление подготовки 010500.62 «Прикладная математика

80

г) , д) .

2) Приведите к каноническому виду квадратичные формы при всевозможных

действительных :

а) , б) , в) , г)

.

3) Докажите, что квадратичная форма положительно определена тогда и только тогда,

когда все корни характеристического многочлена ее матрицы положительны.

4) Квадратичная форма от n неизвестных называется отрицательно определенной, если

ее ранг равен отрицательному индексу инерции и равен числу неизвестных. Докажите,

что квадратичная форма отрицательно определена тогда и только тогда, когда на

любом ненулевом наборе значений переменных принимает отрицательные значения.

ТЕСТЫ

Входящий тест

по дисциплине «Линейная алгебра и аналитическая геометрия», «Алгебра и геометрия»,

«Линейная алгебра», «Аналитическая геометрия»

Геометрия

1. Найти площадь треугольника с вершинами в точках A (-1;1), B (3;5), C (4;1).

1) 20; 2) 10; 3) 16; 4) 8.

2. Найти ординату точки К, лежащей на прямой АВ, если A (-1;-6), B (3;2) и абсцисса

точки К равна 21.

1) -12,5; 2) 12,5; 3) -38; 4) 38.

3. Выбрать верное утверждение.

1)Медианы произвольного треугольника не пересекаются;

2) Медианы произвольного треугольника пересекаются в одной точке и делятся

точкой пересечения пополам;

3) Медианы произвольного треугольника пересекаются в одной точке и делятся

точкой пересечения в отношении 2:1 считая от вершины;

4) Каждая медиана равнобедренного треугольника является одновременно высотой

и биссектрисой.

4. Стороны треугольника относятся как 15:20:25. Каким является треугольник?

1) Тупоугольный; 2) Равносторонний; 3) Равнобедренный; 4) Прямоугольный.

5. Найти длину вектора AВ , если A (-1;2), B (3;-1).

1) 5 ; 2) 13 ; 3) 5; 4) 17 .

6. В трапеции ABCD стороны BC и AD - основания и 2BC =AD. Выразить вектор DB

через векторы DAиCD .

1) DACD2

1 ; 2) DACD

2

1 ; 3) DACD

2

1 ; 4) DACD .

7. В уравнении окружности 8)1()2( 22 yx найти координаты центра и радиус.

1) (2; 1), R= 4; 2) (-2; 1), R= 2 2 ; 3) (2; -1), R= 64; 4) (2; -1), R= 2 2 .

8. В (-7;4;-3). Найти сумму расстояний от точки В до оси Ох и от точки В до плоскости

уОх.

1) 6; 2) 12; 3) 14; 4) 10.

2

332

2

23121

2

1 46322 ххххххххх 2

3

2

23121

2

1 199482 ххххххх

2

221

2

1 23 хххх 2

221

2

1 28 хххх 2

3

2

23121

2

1 6482 ххххххх

4342324132

2

4

2

3

2

2

2

1 522244 хххххххххххххх

Page 81: irbis.amursu.ru · 4 1. J : ; H Q :Я I J H = J : F F : ГЕОМЕТРИЯ И АЛГЕБРА Направление подготовки 010500.62 «Прикладная математика

81

9. Известны координаты вершин треугольника АВС: A (-3;4;2), B (1;-2;5), C (-1;-6;4). ВК –

медиана треугольника. Найти DK.

1) 14 ; 2) 18 ; 3) ;15 4) .10

10. ABCD – параллелограмм: A (4;-1;3), B (-2;4;-5), C (1;0;-4), D (х;у;z). Найти сумму

координат точки D.

1) -3; 2) -5; 3) 6; 4) 4.

11. ABCDA1B1C1D1 - куб. Найти вектор, равный .1111 DСCВАА

1) 11AС ; 2) AC ; 3) BD ; 4) правильного ответа нет.

12. ABCDA1B1C1D1 - куб; .,, 1 cАAbАВаАD

Выразить вектор MK через векторы

.,, cbа

1) ;2

1cbа

2) ;2

1

2

1cbа

3) ;2

1cbа

4) .2

1

2

1cbа

13. Даны координаты точек: A (-3;2;-1), B (2;-1;-3), C (1;-4;3), D (-1;2;-2).

Найти .32 СDАВ

1) 433 ; 2) 521 ; 3) 487 ; 4) 395 .

14. Даны координаты точек: A (3;-2;1), B (-1;2;1), C (2;-3;3), D (-1;1;-2).

Найти косинус угла между векторами CDиАB .

1) 0,75; 2) 0,6; 3) 0,7; 4) 3

2.

15. При каком значении k векторы )9;55;3()2;;6( kbиkkа

перпендикулярны?

1) 2; 2) 3: 3) 2, -3, 6; 4) 3, -2, 4.

16. При каком значении а векторы CDиАB коллинеарны, если A (-2;-1;2), B (4;-3;6), C (-

1;а-1;1), D (-4;-1;а).

1) 1; 2) -2; 3) 2; 4) -1.

17. Дано: .60);̂(,4,4 obabа

Найти ,cos где - угол между векторами

.bиbа

1) 0,07; 2) 15

1; 3)

13

1; 4) 0,08.

18. Диагональ куба равна 12 ед. Найти объем куба.

1) ;3144 3ед 2) ;216ед 3) ;3192 3ед 4) .2216 3ед

19. ABCDA1B1C1D1 - куб. Найти угол между плоскостью, проходящей через точки B, D,

C1, и плоскостью основания.

1) 6

3arccos ; 2)

4

3arccos ; 3) 45

o; 4) 2arctg .

20. Длины диагоналей трех граней прямоугольного параллелепипеда, имеющие общую

вершину, равны смсмсм 53,132,5 . Найти диагональ параллелепипеда.

1) ;73см 2) ;74 см 3) ;61см 4) .27 см

Page 82: irbis.amursu.ru · 4 1. J : ; H Q :Я I J H = J : F F : ГЕОМЕТРИЯ И АЛГЕБРА Направление подготовки 010500.62 «Прикладная математика

82

21. Найти высоту треугольной пирамиды, если все ее боковые ребра по см40 , а

стороны основания равны 10 см, 10 см и 12 см.

1) см4

15; 2) ;2 см 3) ;

2

3см 4) 1,5см.

22. Выбрать уравнение сферы с центром в точке А (2;-1;-4) и радиусом 7.

1) ;7)4()1()2( 222 zyx 2) ;7)4()1()2( 222 zyx

3) ;49)4()1()2( 222 zyx 4) .49)4()1()2( 222 zyx

23. Алюминиевый шар объемом 363см переплавили в равновеликий конус, образующая

которого равна см53 . Найти высоту конуса, если она не более 4 см.

1) см5,2 ; 2) ;10 см 3) ;3 см 4) 32 см.

24. Объем цилиндра равен 633см , а площадь осевого сечения 18

2см

Найти радиус основания цилиндра.

1) см8 ; 2) ;36 см 3) ;9см 4) 7 см.

25. Около куба описан цилиндр, полная поверхность которого равна S. Найти

площадь поверхности куба.

1) ;22 S 2) ;24 S 3) ;)12(

4

S 4)

)12(

6

S.

6.ИТОГОВЫЙ КОНТРОЛЬ

Билеты к экзамену 2 семестр

АмГУ И.о. зав. кафедой

ФМиИ

1 курс _________________

Геометрия и алгебра

2 семестр

Экзаменационный билет №1

1. Вывод общего уравнения плоскости.

2. Линейная оболочка, пересечение и сумма подпространств.

_____________________________________________________________________________

АмГУ И.о. зав. кафедой

ФМиИ

1 курс _________________

Геометрия и алгебра

2 семестр

Экзаменационный билет №2

1. Вывод нормального уравнения плоскости.

2. Квадратичная форма, ее матрица, формы записи. Метод Лагранжа приведения к

каноническому виду.

Итоговый тест

Page 83: irbis.amursu.ru · 4 1. J : ; H Q :Я I J H = J : F F : ГЕОМЕТРИЯ И АЛГЕБРА Направление подготовки 010500.62 «Прикладная математика

83

1. Среди арифметических операций выбрать БАО на множестве 1;0;1

1) сложение, 2) вычитание, 3) умножение, 4) деление.

2. На каких множествах операция вычитания является БАО

1) множество натуральных чисел, 2) множество положительных действительных чисел,

3) множество рациональных чисел, 4) множество нечетных целых чисел.

3. Указать операции, которые не являются коммутативными на указанных множествах

1) baba на множестве Q , 2)

22 baba на множестве R ,

3) abba на множестве R ,4)

5

abba на множестве R .

4. Указать операции на заданных множествах, которые обладают нейтральным элементом

1) bba на множестве Q , 2) 22 bаba на множестве Z ,

3) 2

baba

на множестве R ,4)

3

abba на множестве 0\Q .

5. Указать обратимые операции на данных множествах

1) операция умножения на множестве чисел вида Zbaba ,/5 ,

2) abba на множестве Z ,3) сложение векторов плоскости,

4) умножение на множестве 0\Q .

6. Выяснить, какие множества являются группами относительно указанных операций

1) натуральные числа относительно сложения,

2) множество Q относительно умножения,

3) нечетные целые числа относительно сложения,

4) множество чисел вида 1;1 относительно умножения.

7. Среди данных множеств, указать те, которые являются кольцами относительно

обычных сложения и умножения

1) множество N ,2) множество всех четных целых чисел,3) 1 ,

4) множество чисел вида ,53ba где ba, любые целые числа.

8. Среди данных множеств, указать те, которые являются полями относительно обычных

сложения и умножения

1) множество Z ,2) множество комплексных чисел,3) 0 ,4) множество рациональных

чисел.

9. Результат действий имеет )5(3

41i

i

i

вид:

1) -0,1 -3,7i; 2) 8

3539 i; 3) 0,7 -6,3i; 4)

10

371 i.

10. Решить уравнение 052 zz :

1) 2

1312,1

z ; 2)

2

1912,1

z ; 3)

2

2112,1

iz

; 4)

2

1912,1

iz

.

11. Вычислить 8)22( i :

Page 84: irbis.amursu.ru · 4 1. J : ; H Q :Я I J H = J : F F : ГЕОМЕТРИЯ И АЛГЕБРА Направление подготовки 010500.62 «Прикладная математика

84

1) )4

sin4

(cos16

i ; 2) 256 ; 3) 256; 4) ))4

sin()4

(cos(256

i .

12. Вычислить 3 1 :

1) 1; 2) 1 ; 3) 1, 2

3

2

1i ,

2

3

2

1i ; 4) 1,

2

1

2

3i ,

2

1

2

3i .

13.Записать уравнение с комплексным аргументом, описывающее геометрическое место

точек, соответствующее рисунку

1) 432 iz ; 2) 432 iz ; 3) 4z ; 4) 432 iz .

14. Перестановка (47163285) имеет число инверсий равное:

1) 12; 2) 13; 3) 14; 4) 11.

15. Произведение подстановок

34152

54321

54213

54321 равно:

1) (1); 2)

14235

54321; 3)

24351

54321; 4)

14532

54321.

16. Декремент подстановки

82647531

13572468равен:

1) 6; 2) 5; 3) 0; 4) 7.

17. Определитель 51

43

равен: 1) -19; 2) 19; 3) -9; 4) 11.

18. Определитель

043

101

120

равен:1) 2; 2) -2; 3) -10; 4) 10.

19. Определитель

3211

1123

3121

1132

равен:1) 18; 2) -18; 3) -90; 4) 90.

20. Матрица, обратная к матрице

57

24 имеет вид:

1)

47

25; 2)

47

25; 3)

47

25

6

1; 4)

47

25

34

1.

M

(2;3)

-

1 0

1 2 3 4 5 6 1

2

3

4

5

7

6

x

y

Page 85: irbis.amursu.ru · 4 1. J : ; H Q :Я I J H = J : F F : ГЕОМЕТРИЯ И АЛГЕБРА Направление подготовки 010500.62 «Прикладная математика

85

21. Ранг матрицы

71617

41115

3502

2111

равен:1) 1; 2) 4; 3) 3; 4) 2.

22. Система

84

,23

yаx

ух имеет бесчисленное множество решений при значении

параметра а равном:

1) -12; 2) -4,5; 2) 12; 4) при любом значении а.

23. Исследовать систему уравнений

.18572

,357

,0522

,1328

54321

54321

54321

4321

xxxxx

xxxxx

xxxxx

xxxx

на совместность:

1) система совместна и определена; 2) система совместна и неопределена;

3) система несовместна 4) система имеет только нулевое решение.

24. Среди данных систем векторов укажите те, которые является линейно зависимыми:

1) )2,4,1(1 a , )6,1,3(2 a , )8,3,2(3 a ;

2) )0,0,1(1 a , )0,1,0(2 a , )1,0,0(3 a ;

3) )0,5,4(1 a , )6,9,2(2 a , )5,1;1;5,0(3 a ;

4) )3,7,6(1 a , )8,10,0(2 a , )1,0,0(3 a .

25. Среди данных утверждений укажите верные:

1) линейно независимая система векторов коллинеарные векторы содержать не может;

2) если система векторов содержит компланарные векторы, то система является

линейно независимой;

3) линейно зависимая система векторов нулевой вектор содержать не может;

4) если часть системы векторов линейно зависима, то вся система векторов может

быть линейно независимой.